Sei sulla pagina 1di 51

Imersao Olmpica

Introdu cao `a Teoria dos N umeros


Eduardo Tengan (ICMC-USP)
20 a 23 de janeiro de 2009
Captulo 1
Divisibilidade e Congruencias
Neste captulo, introduzimos dois conceitos fundamentais em Teoria dos N umeros que serao utilizados ao
longo de todo o livro: a rela c ao de divisibilidade e a de congruencia modulo um n umero inteiro.
1.1 Divisibilidade
Um dos conceitos primordiais em Teoria dos N umeros e o da divisibilidade.
Deni cao 1. Dados inteiros dois inteiros d e a, dizemos que
d divide a
ou
d e um divisor de a
ou
a e um m ultiplo de d
e escrevemos
d [ a
se existe um inteiro q tal que a = dq. Em outras palavras, se d ,= 0, dizer que d [ a e o mesmo que dizer que
a fra c ao
a
d
e um inteiro.
Se d n ao divide a (equivalentemente, d n ao e um divisor de a ou a n ao e um m ultiplo de d), escrevemos
d a.
Exemplo 2. Temos que
6 [ 12 mas 12 6;
d [ 4 se, e somente se, d 1, 2, 4;
Um n umero a e par se, e somente se, 2 [ a;
d [ 0, d [ d e d [ d para todo inteiro d;
0 [ 0, j a que existe um inteiro q tal que 0 = 0 q (qualquer inteiro q serve), muito embora a fra c ao
0
0
n ao esteja denida.
As duas propriedades mais importantes da divisibilidade s ao as seguintes:
Teorema 3. 1. (d menor) Se a ,= 0 ent ao
d [ a = [d[ [a[
1
2. (d divide) Dados dois m ultiplos a e b de d, qualquer combina c ao Z-linear de a e b (i.e., uma express ao
da forma ax + by com x, y Z) e um m ultiplo de d. Em smbolos:
_
d [ a
d [ b
= d [ (ax + by)
Demonstra c ao. Se d [ a temos que existe um inteiro q tal que a = dq. Por outro lado, como a ,= 0, temos
tambem que q ,= 0, ou seja, [q[ 1. Logo
a = dq = [a[ = [d[ [q[ [d[ 1 = [d[
o que prova que [a[ [d[.
Para provar o d divide, observe que como a e b sao m ultiplos de d, existem inteiros q
1
e q
2
tais que
a = dq
1
e b = dq
2
. Assim, ax +by = d (q
1
x +q
2
y) e como q
1
x +q
2
y e inteiro, temos que ax +by e tambem
um m ultiplo de d.
Os exemplos a seguir ilustram como aplicar estas duas propriedades para resolver problemas de divisibi-
lidade:
Exemplo 4. Encontre todos os inteiros positivos a e b tais que
1
a
+
1
b
=
1
6
Soluc ao. Isolando a variavel a (por exemplo) temos
1
a
=
1
6

1
b

1
a
=
b 6
6b
a =
6b
b 6
Como queremos a > 0, devemos ter b > 6. Assim, o problema e equivalente a encontrar todos os valores
inteiros b 7 para os quais b 6 [ 6b. Utilizaremos para isto o d divide (neste caso o b 6 divide) para
simplicar o 6b, reduzindo-o a uma constante:
_
b 6 [ 6b
b 6 [ b 6
= b 6 [ 6b 1 + (b 6) (6) b 6 [ 36
Assim, como b 7, temos que b 6 e um divisor positivo de 36, isto e, b 6 1, 2, 3, 4, 6, 9, 12, 18, 36.
Para cada um destes valores, obtemos uma solu c ao com a =
6b
b6
inteiro, de modo que as solu c oes (a, b) sao
(42, 7) (24, 8) (18, 9) (15, 10) (12, 12) (10, 15) (9, 18) (8, 24) (7, 42)
Observe que as solu c oes obtidas sao simetricas em a e b, isto e, se (a, b) e uma solu c ao ent ao (b, a) tambem
e uma solu c ao. Isto n ao e mera coincidencia: a equa cao original ja era simetrica em rela c ao a a e b, logo
suas solu c oes devem manter esta propriedade!
Exemplo 5. Encontre todos os inteiros positivos n tais que
2n
2
+ 1 [ n
3
+ n
2
+ 1
Soluc ao. A estrategia aqui e utilizar o d divide para reduzir o grau da expressao n
3
+n
2
+1 e em seguida
utilizar o d menor. Basta escolhermos uma combina c ao linear conveniente para cancelar o termo de maior
grau n
3
:
_
2n
2
+ 1 [ n
3
+ n
2
+ 1
2n
2
+ 1 [ 2n
2
+ 1
= 2n
2
+ 1 [ 2 (n
3
+ n
2
+ 1) + (n) (2n
2
+ 1)
2n
2
+ 1 [ 2n
2
n + 2
2
Como o grau de 2n
2
n +2 ainda e maior ou igual ao grau de 2n
2
+1, podemos aplicar mais uma vez o d
divide:
_
2n
2
+ 1 [ 2n
2
n + 2
2n
2
+ 1 [ 2n
2
+ 1
= 2n
2
+ 1 [ (2n
2
n + 2) (2n
2
+ 1)
2n
2
+ 1 [ n + 1
Como o grau de n + 1 e estritamente menor do que o grau de 2n
2
+ 1, a ultima rela c ao impoe restri c oes
severas sobre os possveis valores de n, ja que para valores sucientemente grandes de n um polinomio de
maior grau possui valor absoluto maior. Assim, utilizando o d menor, temos dois casos a analisar:
n + 1 = 0 n = 1, que e uma possvel solu c ao, como mostra uma vericac ao direta na rela c ao
de divisibilidade original;
n + 1 ,= 0 e portanto 2n
2
+ 1 [ n + 1 implica que
[2n
2
+ 1[ [ n + 1[ 2n
2
+ 1 n 1 2n
2
n + 2 0
(note que [ n + 1[ = n 1 pois n > 0 e assim n + 1 0). Esta ultima desigualdade n ao possui
solu c oes reais (e muito menos inteiras) ja que o discriminante de 2n
2
n + 2 e = 15 < 0.
Logo a unica solu c ao e n = 1.
Exemplo 6. Encontre todos os inteiros a, b 2 para os quais
ab 1
(a 1)(b 1)
e um inteiro.
Soluc ao. Come camos fazendo uma substitui c ao psicologica (que embora n ao altere em nada o problema,
torna-o aparentemente mais facil, psicologicamente falando, e claro) cuja nalidade e simplicar o deno-
minador da expressao acima:
_
x = a 1
y = b 1

_
a = x + 1
b = y + 1
Assim, o problema e equivalente a encontrar inteiros x, y 1 para os quais a expressao
(x + 1)(y + 1) 1
xy
=
xy + x + y
xy
= 1 +
x + y
xy
e um inteiro. Em outras palavras, devemos encontrar todos os inteiros positivos x, y tais que xy [ x + y.
Como para x e y sucientemente grandes xy e maior do que x + y, novamente nos encontramos na situa c ao
em que o d menor fornece fortes restri c oes para os possveis valores destas variaveis.
Por simetria, podemos nos restringir `as solu c oes em que 1 x y. Neste caso, temos que
xy [ x + y =xy x + y 2y = x 2
Assim, temos dois casos:
x = 1 e assim y [ y + 1, logo y [ (y + 1) y y [ 1 y = 1 pelo y divide.
x = 2 e neste caso 2y [ y + 2, logo 2y [ 2(y + 2) 2y 2y [ 4 y [ 2 e como y x = 2 a unica
possibilidade e y = 2, que claramente e uma solu c ao.
3
Portanto as solu c oes (x, y) sao (1, 1) e (2, 2), isto e, (a, b) = (2, 2) ou (a, b) = (3, 3).
Uma outra maneira de se chegar a estas solu c oes e a seguinte: observe que o quociente
x + y
xy
=
1
x
+
1
y
1 + 1 = 2
ja que x, y 1. Logo, sendo este quociente inteiro, devemos ter
x+y
xy
= 1 ou
x+y
xy
= 2, equa c oes estas que
podem ser resolvidas utilizando a tecnica do d divide, como no exemplo 4.
1.2 Maximo Divisor Comum e Algoritmo de Euclides
Vamos come car relembrando algumas deni c oes que voce provavelmente ja conhece da escola.
Deni cao 7. Se a e b s ao inteiros, com a ,= 0 ou b ,= 0, denotamos por
(a, b)
def
= maxd tais que d [ a e d [ b
(ou mdc(a, b) por enfase) o maior divisor comum de a e b. Denotamos tambem por
mmc(a, b)
def
= minm tais que a [ m e b [ m
o menor m ultiplo comum positivo de a e b.
Note que n ao denimos (0, 0) pois o conjunto dos divisores comuns de 0 e 0 e o Z, logo n ao h a maior
divisor comum!
Exemplo 8. Vejamos alguns exemplos.
Temos que (9, 12) = 3, pois 3 e o maior dentre os divisores comuns de 9 e 12:
1, 3, 9
. .
divisores de 9
1, 2, 3, 4, 6, 12
. .
divisores de 12
= 1, 3
. .
divisores comuns
Da mesma forma, temos que mmc(9, 12) = 36. Note que mdc(9, 12) mmc(9, 12) = 9 12. O fato que
isto n ao e uma coincidencia ser a provado mais adiante no pr oximo captulo.
temos (a, b) = (b, a) > 0 para a e b n ao simultaneamente iguais a 0, pois se d e um divisor comum de
a e b, ele e um divisor comum de b e a (duh!) e, alem disso, d tambem e um divisor comum, sendo
que ou d > 0 ou d > 0;
para qualquer a ,= 0 temos (a, 0) = [a[, pois os divisores comuns de a e 0 s ao simplesmente os divisores
de a, dentre os quais [a[ e o maior.
Deni cao 9. Se (a, b) = 1, dizemos que a e b s ao primos entre si. Em outras palavras, a e b s ao primos
entre si se eles n ao admitem divisores positivos comuns alem do 1.
N ao confunda esta denic ao com a noc ao de n umero primo, aqui a e b n ao precisam ser n umeros
primos!
Ja que tocamos no assunto, vamos aproveitar para denir
Deni cao 10. Um inteiro positivo p e chamado de primo se ele possui exatamente dois divisores positivos,
a saber 1 e p. Um inteiro positvo n e chamado de composto se ele pode ser escrito como produto n = ab de
dois inteiros a e b com 1 < a, b < n.
4
Ao contr ario da crenca popular, 1 n ao e primo. Ele tambem n ao e composto. Ent ao o que ele e?
Ele e uma unidade, ora bolas! Embora possa parecer arbitr ario ` a primeira vista, esta conven c ao
se mostra bastante razo avel na pr atica pois caso admitssemos 1 como primo, v arios teoremas sobre primos
teriam que ser enunciados como . . . v alido para todo primo exceto 1. . .
Exemplo 11. Alguns exemplos.
Os primeiros 10 primos s ao: 2, 3, 5, 7, 11, 13, 17, 19, 23, 29. Uma tabela com os primeiros 240 primos (e
suas razes primitivas) encontra-se no nal do livro;
12 e 35 s ao primos entre si, embora nenhum destes n umeros seja primo;
se p e um n umero primo, como os unicos divisores positivos de p s ao 1 e p, temos que
(p, a) =
_
1 se p a
p se p [ a
dois n umeros consecutivos s ao sempre primos entre si: pelo d divide, um divisor comum d > 0 de n
e n + 1 necessariamente divide 1 = (n + 1) n, logo d = 1;
temos que se d = (a, b) ent ao
a
d
e
b
d
s ao primos entre si, pois se houvesse um divisor comum e > 1
destes dois n umeros ent ao ed seria um divisor comum de a e b estritamente maior do que d.
Agora vamos aplicar o que acabamos de ver para resolver um problema historico: a primeira quest ao da
prova da primeira Olimpda Internacional de Matem atica!
Exemplo 12 (IMO). Mostre que a fra c ao
21n + 4
14n + 3
e irredutvel para todo n natural.
Soluc ao. A fra c ao acima e redutvel se existe um d > 1 que divide simultaneamente o numerador e o
denominador. Assim, mostrar que a fra c ao e irredutvel e o mesmo que mostrar que qualquer inteiro positivo
d que divide simultaneamente 21n+4 e 14n+3 deve ser igual a 1, em outras palavras, devemos mostrar que
21n+4 e 14n+3 sao primos entre si para qualquer n natural. Basta utilizar o nosso velho canivete su co, o
d divide:
_
d [ 21n + 4
d [ 14n + 3
=d [ (2) (21n + 4) + 3 (14n + 3) d [ 1
Como d > 0, devemos ter d = 1, como queramos.
Exemplo 13. Mostre que 2
16
+ 1 e 2
32
+ 1 s ao primos entre si.
Soluc ao. Seja d um divisor comum positivo deste dois n umeros. A ideia natural e tentar utilizar o d
divide, mas a diculdade e encontrar a combina c ao linear correta. Imagine 2
16
+ 1 e 2
32
+ 1 como sendo
polinomios na variavel 2 e tentemos reduzir o grau de 2, como no exemplo 5:
_
d [ 2
16
+ 1
d [ 2
32
+ 1
=d [ 2
16
(2
16
+ 1) (2
32
+ 1) d [ 2
16
1
Como d [ 2
16
+1 e d [ 2
16
1 temos que d divide a diferen ca 2, logo temos em princpio duas possibilidades:
d = 1 ou d = 2. Porem d n ao pode ser igual a 2, pois 2 n ao divide o n umero mpar 2
16
+ 1. Logo d = 1,
como queramos.
5
Calcular o mdc a partir da deni c ao e uma tarefa inconveniente para n umeros grandes (nem t ao grandes
assim, como voce ja deve ter percebido). Felizmente h a um algoritmo bastante eciente para seu c alculo,
baseado no algoritmo da divisao que todos n os estudamos no prim ario! Destes saudosos tempos, sabemos
que dados dois inteiros positivos a e b, o algoritmo da divisao
a b
r q
produz dois inteiros q (quociente) e r (resto) tais que
a = b q + r e 0 r < b
Para quaisquer dois inteiros a e b (possivelmente negativos) com b ,= 0, podemos estender a no c ao de
quociente q e resto r da divisao de a por b utilizando rela c oes analogas `as acima:
Deni cao 14. Dados dois inteiros a e b com b ,= 0, o quociente q e o resto r s ao os unicos dois inteiros
determinados pelas rela c oes
a = b q + r e 0 r < [b[ ()
Denotamos o resto da divis ao de a por b atraves da notac ao a mod b.
Exemplo 15. Temos a seguinte tabela para a = 13 e b = 3:
a b quociente q resto r = a mod b
13 3 4 1
13 3 5 2
13 3 4 1
13 3 5 2
Para que a deni c ao acima realmente fa ca sentido, devemos mostrar
Lema 16. Para quaisquer a e b com b ,= 0, os inteiros q e r na denic ao () acima existem e est ao
unicamente determinados.
Demonstra c ao. Para mostrar a existencia, considere o conjunto
_
. . . , 3[b[, 2[b[, [b[, 0, [b[, 2[b[, 3[b[, . . .
_
dos m ultiplos de [b[ (que e igual ao conjunto dos m ultiplos de b). O inteiro a encontra-se entre dois m ultiplos
consecutivos de [b[, digamos k [b[ a < (k + 1) [b[. Dena
q
def
=
_
k se b > 0
k se b < 0
e r
def
= a k [b[ = a b q
Assim, temos a = b q +r e como k [b[ a < (k +1) [b[ = 0 a k [b[ < [b[ temos tambem 0 r < [b[,
como desejado.
Para demonstrar a unicidade, suponha que exista um outro par de inteiros q

e r

satisfazendo a = bq

+r

e 0 r

< [b[. Assim,


b q + r = a = b q

+ r

b (q q

) = r

r
e portanto r

r e um m ultiplo de b. Porem, como 0 r, r

< [b[, temos que [r

r[ < [b[. Mas o unico


m ultiplo de b com valor absoluto estritamente menor do que [b[ e 0, logo r

r = 0, ou seja, r

= r, e como
() determina q unicamente uma vez conhecidos a, b, r, temos que q = q

, completando a demonstra c ao.


Agora sim estamos prontos para descrever o algoritmo para o c alculo do mdc, o chamado algoritmo de
Euclides ou algoritmo das divis oes sucessivas:
6
Teorema 17 (Algoritmo de Euclides). Sejam a e b inteiros com b ,= 0. Ent ao
(a, b) = (b, a mod b)
Demonstra c ao. Sejam q e r = a mod b o quociente e o resto da divisao de a por b. A m de provar a
igualdade dos mdcs acima, basta mostrar que os pares a, b e b, r possuem o mesmo conjunto de divisores,
pois em particular o maior deles sera igual. Assim devemos mostrar que
_
d [ a
d [ b

_
d [ b
d [ r
Basta utilizar o d divide duas vezes:
(=) Se d divide a e b ent ao d tambem divide r = a bq.
(=) Se d divide b e r ent ao d tambem divide a = bq + r.
Exemplo 18. Aplicando reiteradamente o teorema acima, temos um procedimento eciente para calcular o
mdc de dois inteiros grandes. Por exemplo, temos
(27889, 18937) = (18937, 8952) = (8952, 1033) = (1033, 688) = (688, 345)
= (345, 343) = (343, 2) = (2, 1) = (1, 0) = 1
1.3 O anel Z/mZ dos inteiros modulo m
No estudo da divisibilidade por um inteiro xado m, e conveniente classicar todos os inteiros segundo os
seus restos na divisao euclidiana por m, uma especie de taxonomia de Z. Para isto introduzimos a seguinte
Deni cao 19. Seja m um inteiro xado. Para um inteiro a qualquer, denimos a classe de congruencia de
a m odulo m, denotada por a, como sendo o conjunto de todos os inteiros b que deixam o mesmo resto que
a na divis ao euclidiana por m:
a
def
= b Z [ b mod m = a mod m
= mq + a [ q Z
= b Z [ b a e m ultiplo de m
Mostremos que as varias deni c oes de a acima sao, de fato, todas equivalentes entre si. Escreva a = mq
0
+r
com r = a mod m. Note que os inteiros b que deixam resto r sao exatamente os da forma b = mq

+ r com
q

Z. Assim, b = mq

+(amq
0
) = m(q

q
0
)+a e quando q

percorre todos os inteiros, q = q

q
0
tambem
percorre todos os inteiros (lembre-se de que q
0
e uma constante determinada por a, a saber o quociente de a
por m). Isto mostra a igualdade dos dois conjuntos da primeira e segunda linhas, enquanto que a igualdade
entre os conjuntos da segunda e terceira linhas e imediata.
Exemplo 20. se m = 2, temos
0 = . . . , 4, 2, 0, 2, 4, . . .
1 = . . . , 3, 1, 1, 3, 5, . . .
Assim, temos uma partic ao dos inteiros Z em duas classes, 0 (popularmente conhecidos como pares)
e 1 (vulgo mpares). Assim, no que tange ` a divisibilidade por m = 2, h a somente dois tipos ou
classes de inteiros;
7
No exemplo anterior, temos 0 = 4 = (2) e 1 = 3 = (101), de modo que poderamos indistintamente
escrever a partic ao de Z acima como Z = 0 1 ou Z = 4 (101), por exemplo;
para m = 3, h a tres tipos ou classes de inteiros m odulo 3: os que deixam resto 0, os que deixam
resto 1 e os que deixam resto 2 na divis ao euclidiana por 3.
0 = . . . , 3, 0, 3, 6, . . .
1 = . . . , 2, 1, 4, 7, . . .
2 = . . . , 1, 2, 5, 8, . . .
Em geral, h a precisamente m classes de inteiros m odulo m, agrupados nos conjuntos 0, 1, 2, . . . , m1
segundo os m possveis restos 0, 1, 2, . . . , m1 na divis ao euclidiana por m;
note que, para qualquer m e a, o conjunto a e uma progress ao aritmetica de raz ao m (innita para
ambos os lados) que contem a;
Um fato notavel, e que e a raz ao pela qual introduzimos a deni c ao acima, e a compatibilidade das
opera c oes aritmeticas de soma, diferen ca e produto com a parti c ao de Z em classes modulo m:
Z = 0 1 2 m1
Para entender o que isto signica, vejamos um exemplo para m = 5. Neste caso as classes modulo 5 sao
0 = . . . , 10, 5, 0, 5, 10, . . .
1 = . . . , 9, 4, 1, 6, 11, . . .
2 = . . . , 8, 3, 2, 7, 12, . . .
3 = . . . , 7, 2, 3, 8, 13, . . .
4 = . . . , 6, 1, 4, 9, 14, . . .
Veja que se somarmos um elemento de 2 com outro de 4 sempre obtemos um elemento de 2 + 4 = 6 = 1.
Por exemplo, tome 7 2 e 1 4, temos que 7 + (1) = 6 1. Ou 12 2 e 4 4, temos 12 + 4 = 16 1.
Ou ainda 3 2 e 1 4, temos (3) +(1) = 4 1. Em outras palavras, a parti c ao acima e compatvel
com a soma.
Mais ainda, ela tambem e compatvel com o produto: por exemplo, multiplicando um elemento de 2 com
outro de 4 sempre obtemos um elemento de 2 4 = 8 = 3. Vamos fazer um experimento: tome 3 2 e
9 4, temos que (3) 9 = 27 3 (uau, n ao e que funciona mesmo!).
Chega de experimentos; e hora de provarmos
Lema 21 (Compatibilidade com opera c oes aritmeticas). Fixe um inteiro m. Para quaisquer dois elementos
i a e j b
pertencentes ` as classes de a e b m odulo m, temos que
i + j a + b i j a b i j a b
Em outras palavras, temos
_
i = a
j = b
=
_

_
i + j = a + b
i j = a b
i j = a b
8
Demonstra c ao. Come camos observando que pela deni c ao
_
i a
j b

_
m [ i a
m [ j b
()
Logo, pelo m divide,
m [ (i a) + (j b) m [ (i + j) (a + b) i + j a + b
Analogamente mostra-se que i j a b. Finalmente, novamente pelo m divide temos que () implica
m [ j (i a) + a (j b) m [ i j a b i j a b
O lema acima permite denir a opera c oes com as classes da seguinte forma. Por exemplo, a soma de
duas classes a e b e feita tomando-se qualquer elemento i de a, qualquer elemento j de b e denindo a soma
a + b como sendo a classe que contem i + j. O lema nos garante que esta deni c ao n ao depende de quais
representantes de classe i e j foram escolhidos. Podemos, por exemplo, tomar i = a e j = b, e neste caso
as deni c oes podem ser descritas sucintamente atraves das equa c oes
a + b
def
= a + b, a b
def
= a b, a b
def
= a b
Em particular, observe que para n natural temos
n a
def
= a + a + + a
. .
n vezes
= n a
e
(a)
n
def
= a a . . . a
. .
n vezes
= (a
n
)
Denimos tambem
a
def
= (a)
de modo que (a) + a = a + (a) = 0.
Hora de fazer contas! Mas com isso voce ja contava. . .
Exemplo 22. Temos as seguintes tabelas de adic ao e multiplica c ao para as classes m odulo m = 6:
+ 0 1 2 3 4 5
0 0 1 2 3 4 5
1 1 2 3 4 5 0
2 2 3 4 5 0 1
3 3 4 5 0 1 2
4 4 5 0 1 2 3
5 5 0 1 2 3 4
e
0 1 2 3 4 5
0 0 0 0 0 0 0
1 0 1 2 3 4 5
2 0 2 4 0 2 4
3 0 3 0 3 0 3
4 0 4 2 0 4 2
5 0 5 4 3 2 1
Observe que estas opera c oes nada mais s ao do que as opera c oes usuais nos inteiros, somente que elas s ao
realizadas no ciclo de perodo m = 6:
0
1 2
3
4 5
9
Por exemplo, para somar 4 com 3, fazemos a opera c ao em inteiros 4 + 3 = 7 e depois percorremos o ciclo a
partir do 0 em 7 unidades, obtendo o resultado 1. Naturalmente, isto nada mais e do que uma interpretac ao
pict oria da igualdade 7 = 1, que corresponde a tomar o resto de 7 na divis ao por 6.
Exemplo 23. Calcule, em fun c ao do n umero natural n, o resto de 2
n
na divis ao por 31.
Soluc ao. Em outras palavras, devemos encontrar, em fun c ao de n, a classe de congruencia de 2
n
modulo 31.
Como 2
n
= 2
n
= 2 2
n1
, podemos calcular estas potencias recursivamente, aproveitando o valor anterior:
2
0
= 1
2
1
= 2 1 = 2
2
2
= 2 2 = 4
2
3
= 2 4 = 8
2
4
= 2 8 = 16
2
5
= 2 16 = 32 = 1
2
6
= 2 1 = 2
2
7
= 2 2 = 4
2
8
= 2 4 = 8
2
9
= 2 8 = 16
. . .
E a, percebeu um padr ao? A partir da repeti c ao 2
0
= 2
5
, estabeleceu-se um ciclo de perodo 5. Assim, para
todos os n divisveis por 5 temos 2
n
= 1, para todos os n da forma n = 5k + 1 (ou seja, aqueles que deixam
resto 1 na divisao por 5) temos 2
n
= 2, etc. Portanto a resposta nal e
2
n
mod 31 =
_

_
1 se n e da forma n = 5k
2 se n e da forma n = 5k + 1
4 se n e da forma n = 5k + 2
8 se n e da forma n = 5k + 3
16 se n e da forma n = 5k + 4
Exemplo 24. Calcule o resto da divis ao de 12
2001
por 97.
Demonstra c ao. Nossa missao aqui e determinar a qual das classes de congruencia 0, 1, 2, , 96 modulo 97
o n umero 12
1000
pertence. A estrategia e ir fazendo as contas aos pouquinhos, sempre reduzindo o
resultado.
Temos que uma potencia de 12 pr oxima de 97 e 12
2
= 144. Assim,
12
2
= 144 = 47
Elevando ao quadrado, temos
12
4
= 47
2
= 2209 = 75
Muito bom! Ja sabemos que 12
4
deixa resto 75 na divisao por 97 sem precisar calcular 12
4
explicitamente!
Esta ideia parece promissora. Vamos tentar mais uma vez:
12
8
= 75
2
= (22)
2
= 484 = 96

truque do complementar
Aqui utilizamos um artifcio para simplicar as contas: em vez de utilizar o representante 75 75,
e mais facil calcular o quadrado do igualmente valido representante 22 75 (a quem apelidamos de
complementar de 75 modulo 97 ja que 22+75 = 97). Mas agora, pelo mesmo truque, temos 12
8
= 96 = 1
e nada mais facil do que calcular potencias de 1! (bem, calcular potencias de 1 e um pouco mais facil mas
enm, como diz o velho ditado, 1 dado n ao se olha nos dentes. . . )
Agora rapidamente terminamos o problema: elevando 12
8
= 1 a 250, obtemos 12
2000
= 1. Finalmente
multiplicando por 12, obtemos a resposta nal 12
2001
= 12, isto e, 12
2001
mod 97 = 12.
10
O proximo exemplo mostra como a utiliza c ao de classes de congruencia pode reduzir um problema a
analisar apenas um n umero nito de casos.
Exemplo 25. Mostre que n
5
n e um m ultiplo de 5 para qualquer inteiro n.
Soluc ao. Mostrar que n
5
n e m ultiplo de 5 e mostrar que n
5
n est a na classe de congruencia do 0, isto
e, devemos mostrar que, para qualquer n, n
5
n = 0. Temos
n
5
n = (n
5
) n = (n)
5
n
Mas para n so h a um n umero nito de possibilidades, a saber 0, 1, 2, 3, 4. Assim, o problema, que antes
dispunha sobre um n umero innito de possveis valores para n, agora se reduz a um n umero nito de
verica c oes! Temos (utilizando o truque do complementar 3 = 2 e 4 = 1 para simplicar as contas)
n = 0 = (n)
5
n = (0)
5
0 = 0 0 = 0
n = 1 = (n)
5
n = (1)
5
1 = 1 1 = 0
n = 2 = (n)
5
n = (2)
5
2 = 32 2 = 2 2 = 0
n = 2 = (n)
5
n = (2)
5
(2) =
_
(2)
5
2
_
= 0
n = 1 = (n)
5
n = (1)
5
(1) =
_
(1)
5
1
_
= 0
Logo n
5
n = (n)
5
n e sempre igual a 0, independentemente do valor de n, como desejado.
Terminamos esta se c ao com uma pequena
Deni cao 26. Fixado m, o conjunto
0, 1, 2, . . . , m1
de todas as classes de congruencia m odulo m, juntamente com as opera c oes de soma e produto acima de-
nidos, e chamado de anel de inteiros modulo m, e e denotado por Z/mZ ou simplesmente Z/m.
Em Matem atica, anel e o nome dado a qualquer conjunto A com duas opera c oes binarias + (soma) e
(produto) satisfazendo axiomas que abstraem as propriedades usuais das mesmas opera c oes sobre
os inteiros (por exemplo). Estes axiomas sao
(associatividade da soma) (a + b) + c = a + (b + c) para todo a, b, c A;
(elemento neutro da soma) existe um elemento 0 A tal que a + 0 = 0 + a = a para todo a A;
(inverso da soma) para todo a A existe um b A tal que a + b = b + a = 0;
(comutatividade da soma) a + b = b + a para todo a, b A;
(associatividade do produto) (a b) c = a (b c) para todo a, b, c A;
(elemento neutro do produto) existe um elemento 1 A tal que 1 a = a 1 = a para todo a A;
(distributividade) a (b + c) = a b + a c e (a + b) c = a c + b c para todo a, b, c A.
Um outro exemplo de anel que voce ja pode ter visto e o anel de todas as matrizes n n com entradas
reais, com a soma e o produto usual de matrizes. Aqui, os elementos neutros da soma e do produto sao as
matrizes nula e identidade, respectivamente. Note que o produto neste anel n ao e comutativo.
11
1.4 A Relacao de Congruencia modulo m
A nota c ao a para a classe de congruencia de a modulo m possui o inconveniente de que o modulo m
n ao aparece explicitamente. Alem disso, como vimos nos problemas acima, em muitas quest oes estamos
interessados em apenas decidir se a = b para duas classes a e b. Por isso introduzimos a seguinte
Deni cao 27. Dizemos que a e congruente a b modulo m e escrevemos
a b (mod m)
se, e somente se, a e b pertencem ` a mesma classe de congruencia m odulo m, isto e, se e somente se,
a b (mod m) a = b
a mod m = b mod m
a = mq + b para algum q
m [ (a b)
Em particular, se r = a mod m e o resto da divis ao de a por m, ent ao a r (mod m).
Exemplo 28. Temos que
35 11 (mod 12) pois 12 [ 35 11 (em outras palavras, as classes 35 e 11 de congruencia m odulo
12 s ao iguais). Da mesma forma, 5 19 (mod 12) pois 5 (19) = 24 e um m ultiplo de 12 ou,
equivalentemente, 5 e 19 deixam o mesmo resto 5 quando divididos por 12;
m [ a a = 0 a 0 (mod m).
A nota c ao de congruencia foi introduzida pelo grande matematico alem ao Johann Carl Friedrich Gau
(17771855) em sua obra Disquisitiones Arithmeticae (caso seu Latim esteja enferrujado, a tradu c ao para o
Portugues e Indagac oes Aritmeticas). A deni c ao de congruencias aparece logo no primeiro par agrafo desta
importante obra:
Si numerus a numerorum b, c dierentiam metitur, b et c secundum a congrui dicuntur, sin
minus, ingongrui; ipsum a modulum appellamus. Uterque numerorum b, c priori in casu alterius
residuum, in posteriori vero nonresiduum vocatur.
1
A rela c ao de congruencia tem propriedades muito semelhantes `as da rela c ao de igualdade: quando escre-
vemos a b (mod m), como a = b, em termos de quest oes de divisibilidade por m os inteiros a e b sao por
assim dizer equivalentes. Matem aticos gostam de expressar este fato dizendo que a rela c ao de congruencia
e uma rela c ao de equivalencia.
Deni cao 29. Seja X um conjunto e uma rela c ao bin aria em X. Dizemos que e uma rela c ao de
equivalencia se ela satisfaz os seguintes 3 axiomas:
1. (Reexividade) a a para todo a X.
2. (Simetria) se a b ent ao b a.
3. (Transitividade) se a b e b c ent ao a c.
1
Tradu c ao livre: Se o n umero a divide a diferen ca de dois n umeros b e c, b e c s ao ditos congruentes com relacao `a a, sen ao
incongruentes; a e chamado modulo. Ambos os n umeros b e c s ao chamados de resduos no primeiro caso e de nao resduos no
segundo.
12
Por exemplo, a rela c ao de igualdade em Z e de equivalencia. Outro exemplo: se X e o conjunto de objetos
de uma sala, ent ao a rela c ao
x y o objeto x tem a mesma cor do objeto y
e uma rela c ao de equivalencia (aqui estamos supondo que todos os objetos sao monocrom aticos, ent ao para
n os n ao existe a mesa verde com pes azuis!). Ainda um terceiro exemplo: se X e o conjunto de cartas de
um baralho, dena x y se, e somente se, as duas cartas tem o mesmo naipe.
Os exemplos acima sao casos particulares do seguinte fato: uma parti c ao de X em conjuntos (dois a dois
disjuntos) X
i
dene uma rela c ao de equivalencia em X via x y x e y pertencem ao mesmo conjunto
X
i
. No primeiro exemplo, a rela c ao de igualdade corresponde `a parti c ao de Z em conjuntos unit arios:
Z = 2 1 0 1 2
No segundo exemplo, temos a parti c ao dos objetos segundo suas cores. E no terceiro exemplo, a parti c ao
do baralho em 4 conjuntos segundo seus naipes. Como a rela c ao de congruencia e denida por a b
(mod m) a e b pertencem `a mesma classe de congruencia modulo m, temos que e a rela c ao de
equivalencia em Z correspondente `a parti c ao
Z = 0 1 2 m1
O fato da rela c ao de congruencia ser de equivalencia e interessante, mas ate a nada de impressionante,
anal h a uma multidao de rela c oes de equivalencia pelo mundo afora, qualquer parti c ao de Z daria origem
a uma. O que e notavel, e que foi a grande sacada do Gau, e o fato de que essa rela c ao de equivalencia e
compatvel com as opera c oes aritmeticas usuais. Resumimos esta discuss ao no seguinte
Teorema 30 (Congruencias: propriedades operacionais).
1. (Rela c ao de equivalencia) Para um m xado, a rela c ao de congruencia m odulo m e de equivalencia,
ou seja, satisfaz
(i) (Reexividade) a a (mod m) para todo a.
(ii) (Simetria) se a b (mod m) ent ao b a (mod m).
(iii) (Transitividade) se a b (mod m) e b c (mod m) ent ao a c (mod m).
2. (Compatibilidade com soma, diferenca e produto) Suponha que
a b (mod m) e c d (mod m)
Ent ao
a + c b + d (mod m) (1.1)
a c b d (mod m) (1.2)
a c b d (mod m) (1.3)
Em particular, temos
na nb (mod m) para todo n inteiro (1.4)
a
n
b
n
(mod m) para todo n natural (1.5)
3. (Cancelamento) Se (n, m) = 1 ent ao
na nb (mod m) = a b (mod m)
ou, equivalentemente,
n a = n b = a = b
13
Mas atenc ao, o cancelamento s o vale quando (n, m) = 1!
Demonstra c ao. Ja demonstramos 1 e provaremos 3 no proximo captulo (listamos esta propriedade aqui
por praticidade apenas). Finalmente, 2 e apenas a tradu c ao do lemma 21 na rela c ao de congruencias: por
exemplo, para mostrar 1.1 note que, por hipotese, temos a = b e c = d ent ao
a + c = a + c = b + d = b + d
e assim a + c b + d (mod m).
As propriedades operacionais nos dizem que a rela c ao de congruencia modulo m funciona quase como
se fosse a rela c ao de igualdade, pois podemos somar, subtrair e multiplicar congruencias, e mesmo dividir
em certas situa c oes.
Exemplo 31. Mostre que se n e inteiro positivo mpar ent ao 2
n
+ 1 e um m ultiplo de 3.
Soluc ao. Temos 2 1 (mod 3). Como n e mpar, elevando a n ambos os lados (propriedade 1.5) obtemos
2
n
(1)
n
(mod 3) 2
n
1 (mod 3)
Somando com a congruencia 1 1 (mod 3) (simetria e propriedade 1.1), obtemos
2
n
+ 1 0 (mod 3) 3 [ (2
n
+ 1)
que era o que queramos provar. Veja que, na pratica, o que zemos foi nada mais nada menos do operar
com as congruencias como em uma equa c ao, elevando os dois lados a um mesmo expoente e passando para
o outro lado o 1, como em uma igualdade.
O exemplo acima ilustra o fato de que, ao calcularmos a
n
mod m, e interessante procurar um expoente
d tal que a
d
1 (mod m). No captulo 3 veremos uma formula para tal expoente, mas por hora ca a
mensagem:
1
Exemplo 32. Calcule o resto da divis ao de 2
2002
por 101.
Soluc ao. Aqui, procedemos como em 24. Para calcular 2
2002
mod 101, come camos com uma potencia de
2 pr oxima de 101, por exemplo 2
7
= 128. Elevando ao quadrado varias vezes e utilizando o truque do
complementar, obtemos
2
7
27 (mod 101) = 2
14
729 (mod 101) 2
14
22 (mod 101)
= 2
28
484 (mod 101) 2
28
21 (mod 101)
= 2
56
441 (mod 101) 2
56
64 (mod 101)
Mas 64 = 2
6
, o que nos d a a oportunidade de aplicar nossa nova ferramenta, o cancelamento:
2
56
2
6
(mod 101) = 2
50
1 (mod 101)
E providencialmente encontramos 2
50
1 (mod 101)! Agora e facil terminar: dividindo 2002 por 50,
obtemos 2002 = 50 40 + 2 e assim
2
2002
=
_
2
50
_
40
2
2
= 2
2002
(1)
40
2
2
(mod 101) 2
2002
4 (mod 101),
ou seja, 2
2002
4 (mod 101). Logo o resto da divisao de 2
2002
por 101 e 4.
14
Exemplo 33. Calcule (3
200
1, 3
35
1).
Soluc ao. A ideia natural para calcular o mdc acima e utilizar o algoritmo de Euclides, que e o que vamos
fazer! Primeiramente temos que
(3
200
1, 3
35
1) = (3
35
1, 3
200
1 mod 3
35
1)
e assim devemos calcular 3
200
1 mod 3
35
1. Pela losoa 1 devemos encontar uma potencia de
3 que e congruente a 1 modulo 3
35
1, mas neste caso temos um candidato natural:
3
35
1 (mod 3
35
1)
Dividindo 200 por 35 obtemos 200 = 35 5 + 25, logo
3
200
= (3
35
)
5
3
25
= 3
200
1
5
3
25
(mod 3
35
1)
= 3
200
1 3
25
1 (mod 3
35
1)
Como 0 3
25
1 < 3
35
1, temos que 3
25
1 e o resto da divisao de 3
200
1 por 3
35
1. Agora devemos
calcular
(3
35
1, 3
25
1) = (3
25
1, 3
35
1 mod 3
25
1)
Com o mesmo procedimento acima, obtemos apos algumas contas (verique!) que 3
35
1 mod 3
25
1 =
3
10
1. Assim, aplicando reiteradamente este processo temos
(3
35
1, 3
25
1) = (3
25
1, 3
10
1) = (3
10
1, 3
5
1) = 3
5
1
pois 3
10
1 = (3
5
1)(3
5
+ 1) e m ultiplo de 3
5
1. Logo (3
200
1, 3
35
1) = 3
5
1.
Vejamos um exemplo em que congruencias nos ajudam a provar a inexistencia de solu c oes.
Exemplo 34. Mostre que se n 3 (mod 4) ent ao n n ao pode ser escrito como soma de dois quadrados
perfeitos.
Soluc ao. Suponha que n = x
2
+ y
2
. O ponto crucial aqui e notar que n ao h a muitos quadrados perfeitos
modulo 4. Analisando todas as possibilidades, temos
x = 0 = x
2
= 0
x = 1 = x
2
= 1
x = 2 = x
2
= 0
Assim, um quadrado perfeito so deixa resto 0 ou 1 modulo 4. Portanto
x
2
+ y
2
mod 4 0, 1, 2
Mas como n mod 4 = 3 n ao est a no conjunto acima, n ao podemos ter n = x
2
+ y
2
.
Terminamos com uma simples
Deni cao 35. Um conjunto de m n umeros inteiros
a
1
, a
2
, . . . , a
m
e chamado de sistema completo de resduos (ou restos) se a
i
, a
j
(mod m) para i ,= j, ou seja, se os
n umeros a
i
representam todos os elementos de Z/mZ exatamente uma vez:
Z/mZ = a
1
, a
2
, . . . , a
n

15
1.5 Algumas aplicacoes
Vamos agora mostrar algumas aplica c oes dos conceitos vistos nas se c oes anteriores em problemas que apa-
recem na natureza.
1.5.1 Bases Numericas (vulgo Problemas com Dgitos)
Um dos temas mais populares em olimpadas de Matem atica sao os bons e velhos problemas com dgitos.
O enunciado destes problemas muitas vezes mascaram fatos bem conhecidos da teoria dos N umeros, ent ao e
importante familiarizar-se com eles.
Dado um n umero natural n, podemos expand-lo em base 10 como
n = a
d
10
d
+ a
d1
10
d1
+ + a
1
10 + a
0
()
com dgitos a
i
0, 1, 2, . . . , 9. Por exemplo,
19785 = 1 10
4
+ 9 10
3
+ 7 10
2
+ 8 10
1
+ 5 10
0
Muitos problemas com dgitos se resumem a escrever a expansao (). Por exemplo, os velhos criterios de
divisibilidade por 9 e 10 nada mais sao do que uma aplica c ao imediata do nosso princpio 1 .
Exemplo 36 (criterios de divisibilidade por 9 e 11). Seja
n = a
d
10
d
+ a
d1
10
d1
+ + a
1
10 + a
0
0 a
i
9
a expans ao do n umero natural n na base 10. Ent ao
1. n tem o mesmo resto na divis ao por 9 que a soma de seus dgitos a
0
+ a
1
+ + a
d
. Em particular,
9 [ n 9 [ (a
0
+ a
1
+ + a
d
);
2. n tem o mesmo resto na divis ao por 11 que a soma alternada de seus dgitos a
0
a
1
+ a
2
a
d
(comecando sempre pelo dgito da unidades). Em particular, 11 [ n 11 [ (a
0
a
1
+a
2
a
d
).
Soluc ao. Como 10 1 (mod 9) e 10 1 (mod 11) temos
n a
d
1
d
+ a
d1
1
d1
+ + a
1
1 + a
0
(mod 9)
n a
d
(1)
d
+ a
d1
(1)
d1
+ + a
1
(1) + a
0
(mod 11)
e o resultado segue.
Um problema bastante comum e o de determinar os ultimos dgitos de uma potencia grande. Neste
caso, o fato principal a ser utilizado e o seguinte: suponha que voce queira encontrar os dois ultimos dgitos
a
0
de a
1
de n em (). Como
n = (a
d2
10
d2
+ a
d3
10
d3
+ + a
3
10 + a
2
) 10
2
+ (10a
1
+ a
0
)
e 0 10a
1
+ a
0
10 9 + 9 < 100, temos que 10a
1
+ a
0
e o resto da divisao de n por 10
2
= 100, logo o
problema se resume a calcular n mod 100. Caso voce queira os tres ultimos dgitos, basta calcular n mod 10
3
e assim por diante.
Exemplo 37. Determine os dois ultimos dgitos de 3
2009
.
Soluc ao. Vamos calcular 3
2009
mod 100. Fazendo as contas modulo 100, temos
3
4
= 19 = 3
8
= (19)
2
= 61 = 3
10
= 3
2
61 = 49
Portanto, de 3
10
= 49 obtemos
3
20
= 49
2
= 1 =3
2000
= 1
Ja calculamos 3
8
= 61, logo multiplicando por 3 obtemos 3
9
= 83. Logo 3
2009
= 1 3
9
= 83. Assim, os dois
ultimos dgitos de 3
2009
sao 83.
16
Em alguns problemas, devemos tambem estimar a quantidade de dgitos de n. Isto e simples: como um
n umero n tem d dgitos se, e somente se, ele e maior ou igual a 10
d1
(menor n umero com d dgitos) e
estritamente menor do que 10
d
(menor n umero com d + 1 dgitos), e como o logartmo na base 10 e uma
fun c ao crescente, conclumos que
10
d1
n < 10
d
= d 1 log
10
n < d = d = log
10
n + 1
onde, para um n umero real x, x denota a parte inteira t de x, isto e, o unico inteiro t tal que x1 < t x.
Por exemplo, 2,1 = 2, = 3, 3 = 3 e

2 = 2 (nao e 1!). Resumindo


Lema 38 (N umero de dgitos). Seja n um natural. Ent ao o n umero de dgitos de n e igual a log
10
n +1.
Vejamos como aplicar esta ideia juntamente com as anteriores no seguinte
Exemplo 39. (IMO) Seja A a soma dos dgitos de 4444
4444
e B a soma dos dgitos de A. Ache a soma
dos dgitos de B.
Soluc ao. Este problema pode impressionar `a primeira vista, mas de fato ele e mais simples do que aparenta.
O ponto chave aqui e perceber que a opera c ao soma dos dgitos fazer encolher tremendamente o seu
argumento!
Por exemplo, embora 4444
4444
seja gigantesco, como 4444
4444
< (10
4
)
4444
= 10
17776
, temos que 4444
4444
tem no maximo 17776 algarismos e portanto a soma A de seus dgitos n ao ultrapassa 17776 9 = 159984,
um n umero min usculo quando comparado a 4444
4444
! (sem o fatorial, e claro).
Como A 159984, temos que a soma B de seus dgitos n ao ultrapassa 9 + 9 + 9 + 9 + 9 = 45. De
fato, este e o pior dos casos para A < 10
5
; no caso em que 10
5
A 159999, teramos no maximo
B 1 + 5 + 9 + 9 + 9 + 9 = 42 e 42 < 45.
Assim, B 45 e a soma C dos dgitos de B portanto n ao ultrapassa 3 + 9 = 12. Mas como encontrar
C exatamente? A entram as congruencias: utilizando o criterio de divisibilidade por 9, sabemos que a
opera c ao soma dos dgitos n ao altera a classe de congruencia modulo 9. Como
4444
4444
(4 + 4 + 4 + 4)
4444
(mod 9) 4444
4444
(2)
4444
(mod 9)
e (2)
4444
7 (mod 9) (verique, utilize o fato que (2)
3
1 (mod 9)) conclumos que o resto da divisao
de 4444
4444
por 9, bem como os restos das divisoes de A, B e C por 9, sao todos iguais a 7. Mas 0 < C 12,
logo a unica possibilidade e C = 7, que e a soma procurada.
1.5.2 Usando congruencias para resolver equacoes diofantinas
As congruencias sao extremamente uteis na resolu c ao de equa c oes em inteiros. Tais equa c oes sao chamadas
de diofantinas, em homenagem ao matematico grego Diofanto de Alexandria (oo `o

A ),
que viveu no seculo III D.C. e escreveu uma serie de livros entitulados Arithmetica sobre resolu c oes de
equa c oes algebricas.
Exemplo 40. (OBM) Determine todos os inteiros positivos m e n tais que
m
2
+ 161 = 3
n
Soluc ao. Come camos com um caso particular: suponha que n = 2k seja par. Mas por que estudar este
caso primeiro? Porque agora vale a fatora c ao
m
2
+ 161 = 3
n
161 = 3
2k
m
2
161 = (3
k
m)(3
k
+ m)
Pois e, fatore e fature! H a um n umero nito de maneiras de escrever 161 como o produto de dois inteiros
3
k
m e 3
k
+ m. Utilizando o fato que 161 = 7 23 e que 3
k
m < 3
k
+ m temos somente dois casos:
_
3
k
m = 1
3
k
+ m = 161
e
_
3
k
m = 7
3
k
+ m = 23
17
(na verdade, aqui estamos nos adiantando um pouco porque utilizamos implicitamente o Teorema Funda-
mental da Aritmetica sobre a fatora c ao unica em primos, que sera provado no proximo captulo. . . )
No primeiro caso, resolvendo o sistema nas variaveis m e 3
k
obtemos m = 80 e 3
k
= 81 k = 4,
ou seja, (m, n) = (80, 8). No segundo caso, temos m = 15 e 3
k
= 8, que n ao possui solu c oes inteiras.
Ser a que a solu c ao acima e a unica? Para provar isto, vamos utilizar congruencia para mostrar que n e
obrigatoriamente par. A primeira d uvida que poderia surgir e que modulo vamos usar? Aqui a ideia e
trabalhar modulo um n umero que simplique a equa c ao acima. Por exemplo, podemos tentar apagar o
161, e para isto escolhemos trabalhar modulo um de seus divisores, por exemplo 7:
m
2
+ 161 = 3
n
= m
2
+ 161 3
n
(mod 7) m
2
3
n
(mod 7)
Em outras palavras, agora temos que comparar os possveis restos de m
2
e 3
n
na divisao por 7. Como no
exemplo 23, as potencias de 3 sao periodicas modulo 7 com perodo 6:
1 = 3
0
= 3
6
= 3
12
=
3 = 3
1
= 3
7
= 3
13
=
2 = 3
2
= 3
8
= 3
14
=
6 = 3
3
= 3
9
= 3
15
=
4 = 3
4
= 3
10
= 3
16
=
5 = 3
5
= 3
11
= 3
17
=
Agora veja que h a poucas possibilidades para os quadrados perfeitos m
2
modulo 7; utilizando o truque do
complementar e analisando o n umero nito de casos, temos
m = 0 = m
2
= 0
m = 1 = m
2
= 1
m = 2 = m
2
= 4
m = 3 = m
2
= 2
Assim, para que m
2
= 3
n
, devemos ter n = 6k (quando m
2
= 1), n = 6k +4 (quando m
2
= 4) ou n = 6k +2
(quando m
2
= 2). Em todos os casos, n e par, logo a solu c ao que encontramos acima e a unica da problema.
Exemplo 41. Encontre todos os n umeros naturais x e y tais que 2
x
= 3
y
1.
Soluc ao. Aqui, qual o modulo? Voce lembra do 1 ? Pois ele ataca novamente! Podemos, por
exemplo, trabalhar modulo 4, que e proximo de 3 e e uma potencia de 2:
2
x
= 3
y
1 = 2
x
3
y
1 (mod 4) 2
x
(1)
y
1 (mod 4)
Agora precisamos analisar 2
x
mod 4. A grande maioria das potencias de 2 e divisvel por 4, mas considerar
todas precisamos dividir em casos:
se x = 0, temos 2
0
= 3
y
1 3
y
= 2, o que n ao e possvel;
se x = 1, temos 2
1
= 3
y
1 3
y
= 3 y = 1, logo x = 1 e y = 1 e uma solu c ao;
se x 2 (caso geral), temos 2
x
0 (mod 4). Logo devemos ter (1)
y
1 0 (mod 4), o que ocorre
se, e somente se, y e par. Assim, y = 2a, onde a e natural e voltando na equa c ao original temos uma
fatora c ao:
2
x
= 3
2a
1 2
x
= (3
a
1)(3
a
+ 1)
Como 3
a
1 e 3
a
+ 1 sao divisores de uma potencia de 2, ent ao 3
a
1 e 3
a
+ 1 sao ambas potencias
de 2 pelo Teorema Fundamental da Aritmetica (cuja demonstra c ao veremos mais tarde, n ao perca o
18
proximo captulo!). Mas a diferen ca entre elas e (3
a
+1) (3
a
1) = 2, e as unicas potencias de 2 t ao
pr oximas assim sao 2 e 4. De fato, olhando para as potencias de 2,
2
0
= 1, 2
1
= 2, 2
2
= 4, 2
3
= 8, 2
4
= 16, 2
5
= 32, . . .
observamos que se i > j e i 3 temos 2
i
2
j
2
i
2
i1
= 2
i1
2
2
= 4, logo devemos j i 2 e
agora analisando o n umero nito de possveis pares (i, j) temos que (i, j) = (1, 2) e a unica possibilidade
vi avel.
Consequentemente, 3
a
1 = 2 a = 1 e logo y = 2 e x = 3.
Enm, as unicas solu c oes sao (x, y) = (3, 2) e (x, y) = (1, 1).
1.6 Exerccios
1.6.1 Basicos
1. Encontre todos os inteiros positivos tais que
(a) n + 1 [ n
3
1 (b) 2n 1 [ n
3
+ 1
(c)
1
n
+
1
m
=
1
143
(d) 2n
3
+ 5 [ n
4
+ n + 1
2. Mostre que
(a) 31 [ 20
15
1 (b) 61 [ 20
15
1
(c) 641 [ 2
32
+ 1 (d) 13 [ 2
70
+ 3
70
(e) 7 [ 2222
5555
+ 5555
2222
3. Determine o resto das divis oes de
(a) 4
1234
por 3 (b) 20
100
por 17
(c) 2000 2
2000
1 por 3 (d) 5
20
por 26
4. Encontre os dois ultimos dgitos de
(a) 7
100
(b) 2
2000
(c) 5
600
+ 19
200
(d) 7
2000
2
300
5. Mostre que
_
835
5
+ 6
_
18
1 e divisvel por 112.
6. Mostre que se n e um n umero natural composto, ent ao n e divisvel por um primo p com p

n.
7. Qual o n umero de dgitos de 2008
2008
quando escrito na notac ao decimal? (log
10
2008 3.302763 . . .)
1.6.2 Licao de Casa
8. Mostre que para todo natural n
(a) 7 [ 2
3n+2
+ 3 (b) 7 [ 37
n+2
+ 16
n+1
+ 23
n
(c) 25 [ 72
2n+2
47
2n
+ 28
2n1
(d) 24 [ n(n
2
1)(3n + 2)
9. Mostre que
(a) 2
15
1 e 2
10
+ 1 s ao primos entre si.
(b) 2
32
+ 1 e 2
4
+ 1 s ao primos entre si.
10. Seja a
n
= n
2
+ 100. Determine o maior valor possvel de (a
n
, a
n+1
).
11. Encontrar os 4 ultimos dgitos de 3
2008
na notac ao decimal.
Dica: para reduzir as contas, aplique o bin omio de Newton em (10 1)
1004
.
19
12. Mostre que se m e n s ao dois inteiros positivos tais que mn + 1 e divisvel por 24 ent ao m+ n tambem
e divisvel por 24.
13. Encontre todos os n umeros naturais de dois dgitos divisveis pela soma de seus dgitos.
14. Em cada casa do tabuleiro de xadrez, h a o n umero de gr aos de trigo indicado, como mostra a gura. O
cavalo de Bruno & Bernardo comeca a se movimentar no tabuleiro, de acordo com as regras usuais, a partir
de uma casa qualquer. Quando ele atinge uma casa, come todos os gr aos nela existentes (mas ele n ao come
os gr aos da casa inicial). Quando ele deixa uma casa, n os recolocamos a mesma quantidade de gr aos que
nela existiam. Depois de um certo tempo, o cavalo de Bruno & Bernardo retorna ` a casa inicial e come os
gr aos nela existentes. Prove que o n umero de gr aos que o cavalo de Bruno & Bernardo comeu durante sua
viagem e divisvel por 3
2
63
2
62
2
61

2
16
2
17
2
18

2
15
2
14
2
13
2
12
2
11
2
10
2
9
2
8
2
0
2
1
2
2
2
3
2
4
2
5
2
6
2
7
1.6.3 Questoes de Provas
15. (IMO) Encontre todos os inteiros a, b, c com 1 < a < b < c tais que (a 1)(b 1)(c 1) e divisor de
abc 1.
16. Encontre os dois ultimos dgitos de 2
2n
(2
2n+1
1) para n 1 mpar.
17. Quais os dois ultimos dgitos de 3
3
2009
?
18. (a) Mostre que 81 [ 99 . . . 9
. .
nove noves
.
(b) Determine o menor n umero da forma 99 . . . 9 que e divisvel por 17.
19. Mostre que 41 divide 11 . . . 1 (onde h a 5k dgitos 1, k inteiro positivo).
20. Mostre que 91 divide 11 . . . 1 (onde h a 6k dgitos 1, k inteiro positivo).
21. Mostre que, para todos k, m, n naturais, 5
5k+1
+ 4
5m+2
+ 3
5n
e divisvel por 11.
22. Encontre todos os pares de inteiros positivos (x, y) tais que 2
x
= 1 + 3
y
.
23. Encontre todos os inteiros positivos x e y tais que 3
x
2
y
= 7.
20
Captulo 2
Equacoes Diofantinas Lineares e
Teorema Fundamental da Aritmetica
Uma equac ao diofantina linear (em duas variaveis) e uma equa c ao da forma
ax + by = c
nas variaveis x e y. Diofantina, como voce ja sabe, se refere `a qualquer equa c ao em n umeros inteiros,
enquanto que linear obviamente e uma referencia ao fato de que a equa c ao acima ser a de uma reta no
plano cartesiano.
Assim, resolver uma equa c ao diofantina linear e o mesmo que encontrar todos os pontos inteiros (isto e,
com ambas as coordenadas inteiras) de uma reta. Por exemplo, tomando a reta
6x + 21y = 2
embora haja uma innidade de pontos reais (e ate mesmo racionais) n ao h a nenhum ponto inteiro! De fato,
se x, y Z, o lado esquerdo e m ultiplo de 3, enquanto que o direito n ao.
Neste captulo, vamos aprender a reconhecer quando uma equa c ao diofantina linear possui ou n ao solu c ao
e, em caso armativo, como fazer para encontrar todas. Equa c oes diofantinas lineares tem muitas aplica c oes
importantes: elas permitem provar, por exemplo, os teoremas do cancelamento para congruencias e o Teorema
Fundamental da Aritmetica sobre a decomposi c ao unica em primos.
2.1 Teorema de Bezout
O exemplo acima mostra uma obstru c ao obvia para a existencia de solu c oes inteiras de ax + by = c: se
houver um divisor comum d de a e b que n ao divide c ent ao n ao h a solu c oes inteiras. O teorema de Bezout
diz que, na ausencia de tal obstru c ao, a equa c ao admite solu c ao.
Teorema 42 (Bezout). A equac ao
ax + by = c
admite solu c ao se, e somente se, (a, b) [ c.
Demonstra c ao. Come camos com a implica c ao trivial: suponha que a equa c ao admita solu c ao, isto e, existem
x, y inteiros tais que ax + by = c. Como (a, b) divide o lado esquerdo da equa c ao conclumos que (a, b) [ c
tambem.
Reciprocamente, suponha que (a, b) [ c. Basta mostrar que a equa c ao
ax + by = (a, b)
21
tem solu c ao, pois multiplicando uma solu c ao desta equa c ao por c/(a, b) obtemos uma solu c ao da equa c ao
original. Em outras palavras, devemos mostrar que (a, b) escreve-se como uma combina c ao linear de a e b.
Dentre todas as combina c oes lineares positivas ax+by de a e b (existe pelo menos uma, verique!), seja m
a menor, digamos m = ax
0
+by
0
. Observe que (a, b) [ m pois (a, b) divide a e b e m e uma combina c ao linear
de a e b. Vamos mostrar agora que m divide qualquer combina c ao linear de a e b. Uma vez provado isto,
temos que, em particular, m e um divisor comum de a e b, logo m (a, b), que juntamente com (a, b) [ m,
implica m = (a, b).
Seja t = ax +by uma combina c ao linear qualquer de a e b. Seja q e r o quociente e o resto da divisao de
t por m, de modo que t = qm + r com 0 r < m. Temos
r = t qm = (ax + by) q(ax
0
+ by
0
) = a(x qx
0
) + b(y qy
0
)
Ou seja, o resto r tambem e uma combina c ao linear de a e b! Como r < m, pela minimalidade do m a unica
possibilidade e r = 0, isto e, m [ t, como desejado.
A prova do teorema acima n ao permite encontrar explicitamente uma solu c ao de ax + by = c caso ela
exista. Porem isto pode ser facilmente feito utilizando-se o algoritmo de Euclides! Veja o
Exemplo 43 (Resolvendo Equa c oes Diofantinas Lineares via Euclides). Encontre uma solu c ao inteira de
1001x + 109y = 2.
Soluc ao. Calculamos o mdc (1001, 109) via o algoritmo de Euclides:
1001 = 109 9 + 20
109 = 20 5 + 9
20 = 9 2 + 2
9 = 2 4 + 1
Assim, (1001, 109) = (109, 20) = (20, 9) = (9, 2) = (2, 1) = 1 e pelo teorema de Bezout a equa c ao diofantina
linear acima possui solu c ao.
Para encontrar uma, devemos expressar 2 como combina c ao linear de 1001 e 109. Note que a ultima
divisao permite expressar 1 como combina c ao linear de 9 e 2:
9 1 2 4 = 1
Mas da pen ultima divisao, temos que 2 = 20 9 2, logo substituindo esta expressao na combina c ao
linear acima temos
9 ( 20 9 2) 4 = 1 9 9 20 4 = 1
e agora expressamos 1 como combina c ao linear de 20 e 9! Repetindo este procedimento, eventualmente
expressaremos 1 como combina c ao linear de 1001 e 109; multiplicando tudo por 2, obteremos ent ao nossa
desejada solu c ao!
Vamos fazer isto de modo mais sistem atico: isolamos os restos no algoritmo de Euclides e fazemos as
substitui c oes, partindo da ultima equa c ao, e tomando o cuidado de lembrar quais sao os coecientes nas
equa c oes durante as simplica c oes:
20 = 1001 109 9
9 = 109 20 5
2 = 20 9 2
1 = 9 2 4
22
e assim
1 = 9 ( 20 9 2) 4 = 9 9 20 4
1 = ( 109 20 5) 9 20 4 = 109 9 20 49
1 = 109 9 ( 1001 109 9) 49 = 1001 (49) + 109 450
Portanto (x, y) = (49, 450) e uma solu c ao de 1001x + 109y = 1, logo multiplicando tudo por 2 temos que
(x, y) = (98, 900) e uma solu c ao de 1001x + 109y = 2.
O procedimento acima permite dar uma outra demonstra c ao (da parte n ao trivial) do teorema de Bezout
via o princpio da induc ao nita (PIF). Vamos esbo c a-la aqui quando a b > 0 e (a, b) = 1, deixando para
voce completar os detalhes no caso geral. A indu c ao e sobre mina, b.
Dividindo a por b, temos a = bq + r com 0 r < b. Temos que
ax + by = 1 (bq + r)x + by = 1 b(qx + y) + rx = 1
Como (a, b) = (b, r) = 1 e minb, r = r < b = mina, b, por hipotese de indu c ao podemos encontrar
inteiros x
0
e y
0
para os quais bx
0
+ ry
0
= 1. Logo, da equivalencia acima, temos que x = y
0
e y = x
0
qy
0
e uma solu c ao de ax + by = 1.
O teorema de Bezout n ao serve so para resolver equa c oes diofantinas lineares; ele tem importantes
aplica c oes teoricas, como por exemplo o
Teorema 44 (Alternativa de primo). Se d [ ab e (d, a) = 1 ent ao d [ b. Em particular, se p e um n umero
primo e
p [ a
1
a
2
. . . a
n
ent ao p [ a
i
para algum i.
Demonstra c ao. Pelo teorema de Bezout, existem x, y inteiros tais que ax + dy = 1. Multiplicando por b
temos
abx + bdy = b
Note d divide tanto abx (pois por hipotese d [ ab) como bdy, logo d [ b tambem, provando a primeira parte
do lema.
A segunda parte segue da primeira: se p n ao divide a
1
, ent ao p [ a
2
a
3
. . . a
n
. Agora, suponha que p n ao
divida a
2
tambem. Teramos que p [ a
3
a
4
. . . a
n
. E assim por diante, de modo que enventualmente p ter a
que dividir algum a
i
.
Podemos agora nalmente provar o
Corolario 45 (Cancelamento). Se (n, m) = 1 ent ao
na nb (mod m) = a b (mod m)
ou, equivalentemente,
n a = n b = a = b
Demonstra c ao. Note que
na nb (mod m) m [ n(a b)
Como (n, m) = 1, pela alternativa de primo temos que
m [ a b a b (mod m)
como desejado.
23
Podemos retornar agora `as equa c oes diofantinas lineares. Ja sabemos identicar quando uma equa c ao
destas possui solu c ao e ate mesmo encontrar uma. Como fazer para encontrar todas? Veja o
Exemplo 46. Encontre todas as solu c oes de 1001x + 109y = 2.
Soluc ao. Vamos reescrever o lado direito utilizando a solu c ao particular encontrada no exemplo anterior:
1001x + 109y = 2 = 1001 (98) + 109 900 1001(x + 98) = 109(y 900)
Como 1001 e 109 sao primos entre si, pela alternativa de primo temos que 1001 divide y 900, isto e,
y900 = 1001t para algum t. Assim, substituindo na equa c ao acima, temos 1001(x+98) = 1091001t
x + 98 = 109t, e as solu c oes sao portanto
x = 98 109t e y = 900 + 1001t
com t Z.
Em outras palavras, os pontos inteiros da reta de equa c ao 1001x + 109y = 2 sao os pontos da forma
(x, y) = (98, 900) + (109, 1001)t para t Z, que est ao igualmente espa cados na reta, sendo obtidos a
partir da transla c ao do ponto inteiro (98, 900) por m ultiplos inteiros do vetor (109, 1001).
2.2 Inverso Multiplicativo
Vimos como o cancelamento permite dividir modulo m sob certas circunst ancias. Nesta se c ao, queremos
tornar mais preciso este conceito de divisao modulo m:
Deni cao 47. Dado um elemento a de Z/mZ, denimos o seu inverso multiplicativo, denotado por a
1
,
como sendo o elemento (caso exista) de Z/mZ tal que a a
1
=

1.
Por exemplo, como 7 13 1 (mod 15) temos que 7
1
= 13 em Z/15Z.
Teorema 48. O elemento a Z/mZ possui inverso multiplicativo se, e somente se, a e primo com m.
Demonstra c ao. Note que o inverso multiplicativo x de a existe se, e somente se, a equa c ao
ax 1 (mod m) ax my = 1 para algum y
possui solu c ao. E pelo teorema de Bezout, a equa c ao diofantina linear acima e sol uvel se, e somente se,
(a, m) = 1.
Assim, encontrar o inverso multiplicativo de a Z/mZ e o mesmo que resolver a equa c ao ax my = 1,
o que pode ser feito utilizando-se o algoritmo de Euclides. Assim, por exemplo, temos que 109
1
= 450 em
Z/1001.
De posse do inverso multiplicativo, temos agora todas as ferramentas para realmente tratar congruencias
quase como uma rela c ao de igualdade. Como exemplo, vamos resolver a seguinte equa c ao:
Exemplo 49. Determine todos os inteiros x tais que
13x + 8 6x + 11 (mod 15)
Soluc ao. Isolando a variavel x (i.e. subtraindo 6x + 8 de ambos os lados da congruencia) obtemos
13x + 8 6x + 11 (mod 15) 13x 6x 11 8 (mod 15)
7x 3 (mod 15)
24
Agora gostaramos de dividir por 7 modulo 15. Mas isto e facil! Basta multiplicar pelo inverso de 7 em
Z/15Z, ou seja, 13:
7x 3 (mod 15) 13 7x 13 3 (mod 15)
x 9 (mod 15)
E voila! Temos a resposta: todos os inteiros x tais que x 9 (mod 15), ou seja, todos os inteiros da forma
x = 9 + 15k para k Z.
Vamos ilustrar os conceitos acima provando o
Teorema 50 (Teorema de Wilson). Um natural p > 1 e primo se, e somente se, (p 1)! 1 (mod p).
Demonstra c ao. Suponha que p seja primo. Note que neste caso todos os elementos de Z/pZ, com exce c ao
de 0, sao inversves. Devemos provar que o produto (p 1)! de todos estes inversveis e igual a 1 em Z/pZ.
Vamos ilustar a idea geral da prova com um caso particular primeiro. Por exemplo, para p = 11 podemos
formar parzinhos, cujos produtos sao iguais a 1, isto e, parzinhos da forma a, a
1
:
2, 6 3, 4 5, 9 7, 8
Somente 1 e 10 = 1, que sao seus proprios inversos, cam isolados. Assim, temos
10! = 1 10 = 1
que era o queramos demonstrar.
No caso geral, observe que somente 1 sao seus proprios inversos pois pela alternativa de primo temos
x
2
1 (mod p) p [ x
2
1 p [ (x 1)(x + 1)
p [ x 1 ou p [ x + 1 x 1 (mod p)
Assim, podemos agrupar os demais elementos em pares da forma a, a
1
, de modo que
(p 1)! = 1 (1) = 1
Reciprocamente, suponha agora que p > 1 seja um n umero que satisfa ca (p 1)! 1 (mod p). Ent ao
todo 1 a p 1 e primo como p, ja que a e inversvel modulo p: um inverso e dado por (p 1)!/a (que
e um inteiro!). Porem isto implica que p e primo: se p = ab para 1 < a, b < p ent ao (a, p) = a ,= 1, o que
contradiz o fato anterior.
Corolario 51. Se p e um primo tal que p 1 (mod 4) ent ao a equac ao quadr atica
x
2
1 (mod p)
admite solu c ao.
Demonstra c ao. Basta combinar o teorema de Wilson acima com o truque do complementar. Agrupando a
com a = p a para a = 1, 2, . . . ,
p1
2
, obtemos
(p 1)! (1)
(p1)/2
_
1 2 . . .
p 1
2
_
2
(mod p)
Mas como p 1 (mod 4), temos que (p 1)/2 e par. Logo, pelo teorema de Wilson, a equa c ao acima e
equivalente a
1
_
1 2 . . .
p 1
2
_
2
(mod p)
e assim podemos tomar x = 1 2 . . .
p1
2
.
25
Terminamos esta se c ao com uma importante
Deni cao 52. O conjunto dos elementos inversveis de Z/mZ
(Z/mZ)

def
= a Z/mZ [ (a, m) = 1
e chamado de grupo dos elementos inversveis de Z/mZ.
Note que o produto de dois elementos inversveis a e b e inversvel (o seu inverso e a
1
b
1
), assim
(Z/mZ)

e fechado com rela c ao ao produto.


Exemplo 53. Temos a seguinte tabela de multiplica c ao de (Z/15Z)

:
1 2 4 7 8 11 13 14
1 1 2 4 7 8 11 13 14
2 2 4 8 14 1 7 11 13
4 4 8 1 13 2 14 7 11
7 7 14 13 4 11 2 1 8
8 8 1 2 11 4 13 14 7
11 11 7 14 2 13 1 8 4
13 13 11 7 1 14 8 4 2
14 14 13 11 8 7 4 2 1
Em Matem atica, grupo e o nome emprestado a um conjunto G juntamente com uma opera c ao binaria
(produto) que satisfaz os seguintes tres axiomas:
1. (Associatividade) Para quaisquer a, b, c G, (a b) c = a (b c)
2. (Existencia de elemento neutro) Existe um elemento e G tal que, para todo a G, a e = e a = a
3. (Existencia de inverso) Para qualquer elemento a G existe um elemento a
1
G tal que a a
1
=
a
1
a = e
Se, alem dos tres axiomas acima, o grupo G satisfaz
4. (Comutatividade) Para quaisquer a, b G, a b = b a
ent ao G e chamado de grupo abeliano (em homenagem ao matematico noruegues Niels Henrik Abel (1802
1829)).
2.3 Teorema Fundamental da Aritmetica
Vamos agora provar o Teorema Fundamental da Aritmetica sobre a fatora c ao unica em primos. Embora
velho conhecido de todos n os desde os tempos do ensino fundamental, os livros escolares n ao costumam
incluir uma demonstra c ao deste fato intuitivamente obvio. Entretanto este teorema e longe de ser trivial:
basta apenas mencionar que, embora o resultado fosse conhecido dos matematicos desde os tempos dos
antigos gregos, a primeira demonstra c ao correta foi obtida somente 2000 anos mais tarde por Gau!
Para mostrar que este teorema e algo especial do conjunto de todos os inteiros Z, fa camos um pequeno
experimento mental. Trocando Z pelo conjunto dos pares 2Z, denamos um primo em 2Z da mesma
forma que em Z: um inteiro par e primo se ele n ao pode ser escrito como produto de dois pares. Assim,
2, 6, 10, 14, . . . sao primos enquanto que 4 = 2 2, 8 = 2 4, 12 = 2 6, 16 = 4 4, etc. sao compostos em
2Z. Mas agora
60 = 6 10 = 2 30
sao duas decomposi c oes distintas de 60 em primos!
Muito bem, sem maiores delongas, eis o celebre
26
Teorema 54 (Teorema Fundamental da Aritmetica). Seja n > 1. Ent ao n pode ser escrito como produto
de potencias de primos:
n = p
e1
1
p
e
k
k
onde p
i
denotam primos distintos. Alem disso, esta fatora c ao (dita canonica) e unica a menos de uma
permuta c ao dos primos p
i
.
Demonstra c ao. A demonstra c ao e pelo PIF, sendo que o teorema e claro para n = 2 (base de indu c ao):
sendo 2 primo, ele e a sua propria fatora c ao canonica, que e unica pois em qualquer fatora c ao de 2 os fatores
primos devem ser menores ou iguais a 2, logo so h a uma possibilidade.
Suponha agora que n > 2. Vamos primeiramente mostrar a existencia da fatora c ao. Se n e primo, n ao
h a o que fazer, logo supomos que n e composto, digamos n = ab com 1 < a, b < n. Temos, pela hipotese de
indu c ao, que a e b podem ambos serem escritos como produtos de potencias de primos, logo o mesmo vale
para n = ab.
Agora suponha que tenhamos duas fatora c oes (potencialmente distintas)
n = p
e1
1
p
e
k
k
= q
f1
1
q
f
l
l
()
onde p
i
e q
i
sao primos. Pela alternativa de primo, temos que p
1
divide algum dos q
i
, e renumerando podemos
supor que p
1
[ q
1
. Mas q
1
e primo! Assim, a unica possibilidade e p
1
= q
1
, e cancelando um fator p
1
= q
1
em () obtemos
p
e11
1
p
e
k
k
= p
f11
1
q
f2
2
q
f
l
l
Por hipotese de indu c ao, estas duas fatora c oes sao a mesma a menos da permuta c ao de seus fatores. Assim,
podemos supor k = l, p
i
= q
i
para todo i, e
1
1 = f
1
1 e e
i
= f
i
para i 2. Mas isto implica que as
fatora c oes () tambem sao iguais, completando a demonstra c ao.
Podemos agora obter alguns axiomas vistos na escola:
Corolario 55. Seja
n = p
e1
1
p
e
k
k
a fatora c ao can onica do natural n em primos distintos p
i
. Temos que os divisores naturais de n s ao exata-
mente os n umeros da forma
d = p
f1
1
p
f
k
k
com 0 f
i
e
i
para todo i. Assim, o n umero de divisores positivos de n e igual a
(e
1
+ 1)(e
2
+ 1) (e
k
+ 1)
Demonstra c ao. Temos que todo d = p
f1
1
p
f
k
k
com 0 f
i
e
i
e um divisor de n pois n/d e inteiro.
Reciprocamente, se n = dt, ent ao escrevendo as fatora c oes canonicas de d, t e n e comparando os dois lados,
obtemos que os primos que dividem d e t sao os mesmos que dividem n e, para um primo xado p
i
, os
expoentes f
i
e g
i
em d e t devem satisfazer f
i
+ g
i
= e
i
, logo 0 f
i
, g
i
e
i
. Para mostrar a formula do
n umero de divisores, basta notar que cada f
i
pode ser escolhido independentemente entre e
i
+ 1 possveis
valores 0, 1, . . . , e
i
.
Corolario 56. Sejam
m = p
e1
1
p
e
k
k
n = p
f1
1
p
f
k
k
as fatora c oes can onicas dos naturais m e n em primos distintos p
i
com e
i
, f
i
0 (de modo que podemos
assumir a mesma quantidade de primos nas fatora c oes acima). Temos
mdc(m, n) =

1ik
p
min{ei,fi}
i
e mmc(m, n) =

1ik
p
max{ei,fi}
i
e portanto
mdc(m, n) mmc(m, n) = m n
27
Demonstra c ao. As formulas para o mdc e mmc decorrem da descri c ao dos divisores do corol ario anterior,
enquanto que a ultima formula e consequencia do fato de que, para todo i, maxe
i
, f
i
+ mine
i
, f
i
=
e
i
+ f
i
.
A prop osito, a ultima formula do corol ario anterior fornece um metodo eciente para o c alculo do mmc
de dois n umeros: calcule primeiro o mdc via algoritmo de Euclides e depois fa ca mmc(m, n) = mn/(m, n).
Para n umeros grandes, fatorar em primos e uma opera c ao muito mais custosa do que a que acabamos de
descrever!
Vejamos algumas aplica c oes dos resultados acima.
Exemplo 57. Seja n um inteiro positivo que n ao e um quadrado perfeito. Ent ao

n e um n umero irracional.
Soluc ao. Seja n = p
e1
1
p
e
k
k
a fatora c ao canonica de n em potencias de primos p
i
distintos. Se todos os
e
i
s fossem pares, n seria um quadrado perfeito, logo existe um e
i
que e mpar, digamos e
1
.
A demonstra c ao de que

n e irracional e por redu c ao ao absurdo. Suponha que

n =
a
b
com a, b inteiros
positivos. Ent ao
a
2
= nb
2
()
Pelo teorema fundamental da aritmetica, a potencia de p
1
em ambos os lados de () deve ser igual. Se p

1
e a
potencia de p
1
na fatora c ao canonica de a, ent ao p
2
1
sera a potencia de p
1
na fatora c ao canonica de a
2
, isto
e, uma potencia com expoente par. Analogamente, o expoente da potencia de p
1
na fatora c ao canonica de
b
2
tambem e par, digamos 2, e como a potencia de p
1
na fatora c ao de n e p
e1
1
temos que a potencia de p
1
na fatora c ao do lado direito de () e 2 + e
1
, um n umero mpar. Logo as potencias de p
1
do lado esquerdo
e direito de () n ao podem ser iguais, uma contradi c ao.
Exemplo 58. Se (m, n) = 1 e mn e uma k-esima potencia perfeita, ent ao m e n s ao ambos k-esimas
potencias perfeitas.
Soluc ao. Temos mn = x
k
para algum x. Se p
t
e a potencia do primo p na fatora c ao canonica de x, temos
que p
kt
e a potencia de p na fatora c ao canonica de mn. Porem, como (m, n) = 1, temos que p ou divide m
ou divide n, mas n ao ambos; digamos que p [ m. Desta forma p
kt
e a potencia de p na fatora c ao canonica
de m. Como o raciocnio anterior e valido para qualquer primo, temos que m e n sao ambos produtos de
potencias de primos cujos expoentes sao m ultiplos de k, logo sao ambos k-esimas potencias perfeitas.
Observe que se (m, n) ,= 1 a conclusao n ao e valida! Por exemplo, temos que nem m = 3
1
e nem
n = 3
3
sao quadrados perfeitos, mas mn = 3
4
e um quadrado perfeito! Na situa c ao acima, quando
(m, n) = 1, isto n ao ocorre pois n ao e possvel quebrar a potencia p
kt
entre m e n.
O fato anterior e um fato extremamente util na resolu c ao de equa c oes diofantinas. Veja o
Exemplo 59. Determine todas as solu c oes de
x
2
+ y
2
= z
2
com x, y, z inteiros positivos dois a dois primos entre si.
Soluc ao. Note que temos uma fatora c ao (fatore e fature!)
x
2
= z
2
y
2
= (z y)(z + y)
Ser a que (z y, z + y) = 1? Em caso armativo, poderamos utilizar o exemplo acima para concluir que
z y e z + y sao ambos quadrados perfeitos. Seja d = (z y, z + y). Pelo d divide, temos que d divide a
soma 2z e a diferen ca 2y destes dois fatores. Porem, como (z, y) = 1 por hipotese, temos que d [ 2. Assim
temos que analisar a paridade destes termos.
28
Como x, y, z sao primos entre si, nem todos eles podem ser pares. Logo h a exatamente dois mpares e um
par. Por outro lado, analisando modulo 4 como no exemplo 34, temos que n ao podemos ter x e y mpares
e z par, pois neste caso x
2
+ y
2
2 (mod 4) enquanto que z
2
0 (mod 4). Logo z e mpar e por simetria
podemos assumir que x e mpar e y e par. Logo z y e z + y sao mpares e assim d = 1.
Logo pelo exemplo anterior, temos que existem naturais m e n mpares com m < n tais que x = mn,
z y = m
2
e z + y = n
2
. Resolvendo o sistema, obtemos
x = mn y =
n
2
m
2
2
z =
n
2
+ m
2
2
()
S o falta determinar condi c oes para que x, y, z sejam dois a dois primos entre si. Note que qualquer fator
primo comum a dois dentre os inteiros x, y, z deve dividir o terceiro, logo este n umeros serao dois a dois
primos entre si se, e somente se, (y, z) = 1 (por exemplo). Temos (c.f. demonstra c ao do algoritmo de
Euclides)
(y, z) =
_
n
2
m
2
2
,
n
2
+ m
2
2
_
=
_
n
2
m
2
2
+
n
2
+ m
2
2
,
n
2
+ m
2
2
_
=
_
n
2
,
n
2
+ m
2
2
_
()
=
_
n
2
, n
2
+ m
2
_
= (n
2
, m
2
)
onde () utiliza o fato de que n
2
e mpar, de modo que multiplicar a segunda entrada por 2 n ao altera o mdc.
Assim, (y, z) = 1 (m
2
, n
2
) = 1 (m, n) = 1 (utilize a expressao do mdc em termos da fatora c ao
de m e n).
Em, a resposta nal e dada por () para m > n naturais mpares primos entre si.
Conclumos esta se c ao com uma
Deni cao 60. Seja p um primo. Dizemos que p

divide exatamente m (em smbolos p

| m) se e o
expoente de p na fatora c ao can onica de m (ou, equivalentemetne, se p

e a maior potencia de p que divide


m).
Eis uma aplica c ao classica:
Teorema 61 (Fatores do Fatorial). Seja p um primo. Sendo
=
_
n
p
_
+
_
n
p
2
_
+
_
n
p
3
_
+
temos que p

| n!
Observe que a soma acima e nita pois os temos
_
n
p
i
_
sao eventualmente iguais a zero. E relembrando:
para um n umero real x, x denota a parte inteira t de x, isto e, o unico inteiro t tal que x 1 < t x.
Demonstra c ao. No produto n! = 1 2 . . . n, apenas os m ultiplos de p contribuem com um fator p. H a
_
n
p
_
tais m ultiplos entre 1 e n. Destes, os que sao m ultiplos de p
2
contribuem com um fator p extra e h a
_
n
p
2
_
tais fatores. Dentre estes ultimos, os que sao m ultiplos de p
3
contribuem com mais um fator p e assim por
diante, resultando na formula acima.
Exemplo 62. Determine com quantos zeros termina 100!
Soluc ao. O problema e equivalente a determinar a maior potencia de 10 que divide 100! Como h a muito
mais fatores 2 do que 5 em 100!, temos que determinar apenas a maior potencia de 5 que divide 100! e agora
e so aplicar a formula anterior: temos 100/5 + 100/5
2
= 24, logo a maior potencia de 5 que divide 100!
e 5
24
. Portanto 100! termina em 24 zeros.
29
2.4 Teorema Chines dos Restos
Vamos agora considerar o problema de resolver sistemas de congruencias lineares. Vejamos um
Exemplo 63. Determine o menor inteiro positivo x que deixa restos 1 e 13 nas divis oes euclidianas por
109 e 1001, respectivamente.
Soluc ao. Devemos encontrar a menor solu c ao positiva do sistema
_
x 1 (mod 109)
x 13 (mod 1001)
Da primeira congruencia temos que x = 1 + 109k para algum k Z. Substituindo na segunda, obtemos
portanto
1 + 109k 13 (mod 1001) 109k 12 (mod 1001) ()
Agora, multiplicando pelo inverso multiplicativo de 109 m odulo 1001, que e igual a 450, temos
() 450 109k 450 12 (mod 1001) k 395 (mod 1001)
Portanto k = 395 + 1001t e assim
x = 1 + 109 (395 + 1001t) x = 43056 + 109 1001t
x 43056 (mod 109 1001)
e o menor inteiro positivo satisfazendo estas condi c oes e 43056.
Em muitas aplica c oes, n ao e necessario obter as solu c oes explicitamente, apenas saber se elas existem.
Para isto, temos o importante
Teorema 64 (Teorema Chines dos Restos). Sejam m
1
, m
2
, . . . , m
r
inteiros positivos que s ao primos entre
si, dois a dois, e sejam a
1
, a
2
, . . . , a
r
inteiros quaisquer. Ent ao o sistema de conguencias
x a
1
(mod m
1
)
x a
2
(mod m
2
)
.
.
.
x a
r
(mod m
r
)
admite uma solu c ao x, unica m odulo m = m
1
m
2
. . . m
r
.
Apresentamos duas demonstra c oes para este teorema, que ilustram tecnicas distintas.
Primeira prova. Considere a fun c ao diagonal
Z/mZ Z/m
1
Z Z/m
r
Z
a (a, . . . , a)
Primeiramente, precisamos mostrar que esta fun c ao est a bem denida, isto e, que escolhendo outro repre-
sentante de classe b a em Z/mZ temos b = a em cada Z/m
i
Z (cuidado, as barras denotam redu c ao
modulo diferentes n umeros!) Mas isto decorre facilmente do fato de m
i
[ m, pois b = a + km para algum
k Z de modo que b a (mod m
i
) para todo i.
O teorema e equivalente a mostrar que esta fun c ao e sobrejetora (isto e, que existe um x Z/mZ que
atinge qualquer tupla (a
1
, . . . , a
r
) Z/m
1
Z Z/m
r
Z. Como o n umero de elementos em ambos os
30
lados e igual a m = m
1
m
2
. . . m
r
, basta mostrar que esta fun c ao e injetora. Se dois elementos a, b Z/mZ
tem mesma imagem, i.e.,
(a, . . . , a) = (b, . . . , b) a b (mod m
i
) para todo i
temos que m
i
[ a b para todo i. Mas como os m
i
sao dois a dois primos entre si, temos (olhando para a
fatora c ao em primos) que m [ a b, ou seja, a = b em Z/mZ, que era o que queramos mostrar.
Segunda prova. Observe que como (m
i
, m
j
) = 1 para i ,= j, temos que
_
m
mj
, m
j
_
= 1 (note que
m
mj
e um
inteiro!) enquanto que
m
mj
0 (mod m
i
) para i ,= j. Seja b
j
um inverso multiplicativo de
m
mj
modulo m
j
(que existe pois
m
mj
e m
j
sao primos entre si). Tome
x =
m
m
1
b
1
a
1
+ +
m
m
r
b
r
a
r
M odulo m
i
, temos
x
0

m
m
i
b
i
a
i
(mod m
i
) x
0
a
i
(mod m
i
)
Ent ao x
0
e uma solu c ao do nosso sistema. Agora se x
0
e x
1
sao duas solu c oes temos x
0
x
1
(mod m
i
) para
cada i. Como (m
i
, m
j
) = 1 para i ,= j temos como na primeira solu c ao que x
0
x
1
(mod m), mostrando a
unicidade.
Exemplo 65. Mostre que existem 1000 inteiros consecutivos compostos.
Soluc ao. Uma maneira de garantir que os inteiros consecutivos x+1, x+2, . . . , x+1000 sejam compostos e
torna-los m ultiplos de primos xos, com o cuidado de escolher x grande o suciente para que nenhum destes
n umeros coincida com um destes primos.
Assim, escolha 1000 primos distintos p
1
, . . . , p
1000
e considere o sistema
x 1 (mod p
1
)
x 2 (mod p
2
)
.
.
.
x 1000 (mod p
1000
)
Pelo Teorema Chines dos Restos, o sistema acima possui innitas solu c oes, logo basta tomar uma solu c ao x
sucientemente grande.
2.5 Exerccios
2.5.1 Basicos
24. Resolva as seguintes equac oes diofantinas lineares:
(a) 172x + 13y = 1 (b) 233x + 144y = 1
(c) 123x + 130y = 13 (d) 280x + 49y = 2
(e) 391x + 1377y = 34 (f ) 22x + 121y = 11
25. Mostre que se x e y s ao tais que N = (x + 6y)(2x + 5y)(3x + 4y) e divisvel por 7 ent ao N e m ultiplo
de 343.
26. Encontre o inverso multiplicativo (ou mostre que ele n ao existe):
(a) 7 mod 13 (b) 12 mod 143
(c) 22 mod 121 (d) 19 mod 200
31
27. Com quantos zeros termina 2008! quando escrito na notac ao decimal?
28. Prove o seguinte criterio de divisibilidade por 7: dado um inteiro n, seja d o ultimo dgito de n e m o
n umero obtido a partir de n apagando-se o ultimo dgito d; ent ao n e divisvel por 7 se, e somente se, m2d
e divisvel por 7. Por exemplo, para n = 8638, temos 863 2 8 = 847 e 84 2 7 = 70, que e divisvel por
7, logo 8638 e divisvel por 7 tambem.
29. Sejam m
0
, m
1
, . . . , m
r
inteiros positivos que s ao dois a dois primos entre si. Mostre que existem r + 1
inteiros consecutivos s, s + 1, . . . , s + r tal que m
i
[ s + i para i = 0, 1, . . . , r.
30. Existem 2009 inteiros consecutivos tal que cada e divisvel por uma centesima potencia de um primo?
2.5.2 Licao de Casa
31. (Est onia) Determine todos os restos possveis da divis ao do quadrado de um n umero primo com 120 por
120.
32. Seja n um n umero natural e seja
n = p
e1
1
p
e
k
k
a fatora c ao can onica de n em potencias de primos p
i
distintos. Mostre que a soma dos divisores positivos
de n e igual a
p
e1+1
1
1
p
1
1

p
e2+1
2
1
p
2
1
. . .
p
e
k
+1
k
1
p
k
1
2.5.3 Questoes de Prova
33. Um ponto inteiro (x, y) Z
2
e visvel se (x, y) = 1. Existe um ponto (a, b) Z
2
cuja dist ancia a todo
ponto visvel e pelo menos 1000?
34. (IMO) Encontre um par de inteiros positivos a e b tais que
(i) ab(a + b) n ao e divisvel por 7;
(ii) (a + b)
7
a
7
b
7
e divisvel por 7
7
.
35. Seja p(x) um polin omio n ao constante com coecientes inteiros e seja k um inteiro qualquer. Prove que
existe um inteiro m tal que p(m) tem pelo menos k fatores primos distintos.
32
Captulo 3
Teorema de Euler-Fermat
Conforme vimos, para calcular uma potencia a
d
mod m, e interessante obter algum expoente n tal que
a
n
1 (mod m) (nossa velha losoa 1 lembra-se?). O teorema de Euler-Fermat mostra como
encontrar este expoente magico n > 0 de modo que a
n
1 (mod m).
Primeiramente, vejamos os casos em que podemos esperar a existencia de tal expoente magico. Se
d = (a, m) ,= 1, tal expoente n ao pode existir, pois caso contrario teramos
a
n
1 (mod m) a
n
= mx + 1 para algum x
Porem, d divide a e m, logo d divide 1 = a
n
mx, uma contradi c ao.
No caso em que a e m sao primos entre si, a sim podemos esperar encontrar tal expoente magico. De
fato, em Z/mZ considere as potencias de a
1, a, a
2
, a
3
, a
4
, . . .
Como Z/mZ e um conjunto nito, esta lista contem (muitos) elementos repetidos. Se i > j sao tais que
a
i
= a
j
, como a e m sao primos entre si, temos que a e inversvel em Z/mZ (ou seja, a (Z/mZ)

) e desta
forma
a
i
= a
j
a
ij
= 1 a
ij
1 (mod m)
o que mostra a existencia do expoente magico n = i j > 0.
Melhor do que saber que ele existe, e saber calcula-lo.

E para isto que temos o teorema de Euler-Fermat.
3.1 Funcao de Euler
Deni cao 66. Seja n um inteiro positivo. A fun c ao de Euler e denida como
(m)
def
= quantidade de inteiros 1 d m tais que d e m s ao primos entre si
= n umero de elementos do grupo (Z/mZ)

A fun c ao de Euler foi introduzida, advinhem, pelo grande matematico su co Leonhard Paul Euler
(17071783), em conex ao com o teorema de Euler-Fermat que discutiremos a seguir. Esta fun c ao tambem
e conhecida em Ingles por Euler totient function, nome dado por James Joseph Sylvester (18141897),
matematico britancio que adorava inventar novas palavras complicadas.
Exemplo 67. Eis alguns exemplos:
(1) = 1;
33
(12) = 4 pois (Z/12Z)

= 1, 3, 5, 7 tem 4 elementos;
temos que
p e um n umero primo (p) = p 1
De fato, se p e primo, ent ao dentre os n umeros 1, 2, . . . , p, todos s ao primos com p com excec ao do
pr oprio p. Por outro lado, se p = ab e composto com 1 < a, b < p, ent ao na lista 1, 2, . . . , p, os n umeros
a, b e p n ao s ao primos com p, logo (p) < p 1 neste caso. Finalmente, se p n ao e primo e nem
composto (ent ao o que ele e?), temos que p = 1 (ah, e verdade, 1 n ao e nem primo nem composto!)
logo (p) = 1 ,= p 1 = 0.
Calcular diretamente atraves da deni c ao pode n ao ser muito pratico, ent ao vamos provar
Lema 68 (Calculando ). Seja
m = p
1
1
p
2
2
. . . p

k
k
a fatora c ao can onica de m em potencias de primos distintos p
i
. Ent ao
(m) = (p
1
1
p
11
1
) (p
2
2
p
21
2
) . . . (p

k
k
p

k
1
k
)
= m
_
1
1
p
1
_

_
1
1
p
2
_
. . .
_
1
1
p
k
_
Demonstra c ao. Seja
Q
def
= probabilidade de que um n umero 1 d m seja primo com m
=
(m)
m
Vamos calcular esta probabilidade de outra maneira. Temos que d e primo com m se, e somente se, d n ao e
m ultiplo de nenhum fator primo p
i
que divide m. A probabilidade de que d seja m ultiplo de p
i
e
1
pi
ja que h a
p
i
possveis restos igualmente provaveis para d mod p
i
. Assim, a probabilidade de que d n ao seja m ultiplo
de p
i
e 1
1
pi
. Portanto a probabilidade Q de que d n ao seja m ultiplo de nenhum p
i
e
Q =
_
1
1
p
1
_

_
1
1
p
2
_
. . .
_
1
1
p
k
_
Comparando as duas formulas para Q, obtemos o resultado.
Exemplo 69. Pela f ormula temos
(12) = 12
_
1
1
2
__
1
1
3
_
= 4
que, incrivelmente, coincide com o valor que encotramos anteriormente!
Utilizando a formula acima, e facil provar o corol ario a seguir, que deixamos como exerccio para voce:
Corolario 70. A fun c ao e multiplicativa, isto e, para (m, n) = 1 temos
(m n) = (m) (n)
Esta rela c ao s o vale quando m e n s ao primos entre si!
3.2 Teorema de Euler-Fermat
Agora estamos prontos para o teorema propriamente dito! Come camos enunciando o teorema e suas variantes.
Em seguida, esbo camos a sua prova num caso particular, mas que ja encerra todos os ingredientes principais
da demonstra c ao no caso geral, que e atacado logo em seguida.
34
3.2.1 Enunciado do teorema e suas variantes
Teorema 71 (Euler-Fermat). Se (a, m) = 1, ent ao
a
(m)
1 (mod m)
Equivalentemente, se a (Z/mZ)

ent ao
a
(m)
= 1
Trocando em mi udos: se a base a e prima com o modulo m, ent ao existe um expoente, a saber (m), tal
que a
(m)
1 (mod m). Note que uma vez encontrado um tal expoente magico, h a innitos deles: para
qualquer k 0 temos automaticamente
a
k(m)
1 (mod m)
Exemplo 72. (Para os ceticos) Tome a = 3 e m = 14. Temos (14) = 6. Fazendo as contas na ra ca:
3
3
1 (mod 14) = 3
6
1 (mod 14)
Incrvel! N ao e que funciona mesmo. . .
O teorema, no formato acima, foi demonstrado por Euler em 1736. Um caso particular ja havia sido
(de)monstrado pelo grande matematico frances Pierre de Fermat (16011665) quase um seculo antes, a saber
o caso em que m = p e um primo. Neste caso, temos (p) = p 1, logo
a
p1
1 (mod p)
para todo a primo com p (isto e so um jeito complicado de dizer que a n ao e um m ultiplo de p). Agora
vamos multiplicar ambos os membros da congruencia por a. Temos
a
p
a (mod p) ()
Pensemos mais um pouco. Temos que () so poderia falhar se a n ao fosse primo com p, isto e, so se
p [ a, o que na linguagem das congruencias e o mesmo que a 0 (mod p). Ora, substituindo este valor em
(), obtemos uma rela c ao verdadeira ainda. Assim () vale para todo inteiro a. Obtemos como corol ario o
resultado original de Fermat:
Teorema 73 (Pequeno Teorema de Fermat). Seja p um primo. Ent ao para todo inteiro a temos
a
p
a (mod p)
O teorema acima admite uma prova combinatoria bem simples e interessante. Suponha que desejamos
formar colares com p contas e temos contas de a cores distintas. De quantas maneiras podemos montar estes
colares, sendo que dois colares sao considerados o mesmo se um pode ser obtido a partir do outro por meio
de uma rota c ao?
Se n ao houvesse rota c oes envolvidas, a resposta seria apenas o n umero de sequencias (a
1
, a
2
, . . . , a
p
)
onde a
i
representam uma dentre a cores, ou seja, o n umero de colares seria a
p
. Com as rota c oes, cada colar
distinto pode corresponder a mais de uma sequencia (a
1
, a
2
, . . . , a
p
) diferente. Temos dois casos a analisar:
Todas as contas tem a mesma cor. Neste caso h a a colares.
O colar n ao e monocrom atico. Neste caso, cada colar d a origem, por rota c ao, a exatamente p sequencias
(a
1
, a
2
, . . . , a
p
)
(a
2
, a
3
, . . . , a
p
, a
1
)
(a
3
, a
4
, . . . , a
p
, a
1
, a
2
)
.
.
.
(a
p
, a
1
, a
2
, . . . , a
p1
)
35
De fato, vamos mostrar que se duas das sequencias acima sao iguais ent ao o colar e monocrom atico.
Suponha por exemplo que (a
1
, a
2
, . . . , a
p
) = (a
i+1
, a
i+2
, . . . , a
i+p
) para algum 0 < i < p (onde os
ndices sao tomados m odulo p), isto e, a sequencia (a
1
, a
2
, . . . , a
p
) e igual ao seu i-esimo shift.
Ent ao teramos
a
1
shift
= a
i+1
shift
= a
2i+1
shift
= a
3i+1
shift
= a
4i+1
shift
=
(onde novamente os ndices sao tomados m odulo p). Note que basta mostrar que os n umeros 1, i +
1, 2i + 1, 3i + 1, . . . , (p 1)i + 1 percorrem todos os inteiros modulo p (ou seja, que eles formam um
sistema completo de resduos modulo p pela deni c ao 35). Se houvesse uma repeti c ao ri + 1 si + 1
(mod p), como i e inversvel modulo p (0 < i < p), teramos r s (mod p). Mas como 0 r, s < p,
isto implica r = s, ou seja, a repeti c ao ocorreu porque come camos como o mesmo elemento! Logo os
p n umeros 1, i + 1, 2i + 1, 3i + 1, . . . , (p 1)i + 1 realmente formam um sistema completo de restos,
terminando a prova.
O total de colares e portanto a soma dos monocrom aticos e dos policromaticos, isto e,
a +
a
p
a
p
Mas este total e um n umero inteiro (anal de contas, estamos contando n umero de colares)! Logo
a
p
a
p
e
um n umero inteiro, isto e, a
p
a (mod p), and thats Fermat for you!
O teorema de Fermat tem uma consequencia fant astica, o sonho de todo aluno que estuda aritmetica na
escola!
Corolario 74 (O sonho de todo estudante). Se p e um primo ent ao
(a + b)
p
a
p
+ b
p
(mod p)
para quaisquer inteiros a e b.
Demonstra c ao. De fato, aplicando o pequeno teorema de Fermat tres vezes temos
_

_
(a + b)
p
a + b (mod p)
a
p
a (mod p)
b
p
b (mod p)
Assim, modulo p, (a + b)
p
a + b a
p
+ b
p
.
3.2.2 Um esboco da demonstracao
Vamos ver um caso particular: fa camos a = 3 e m = 14. Vamos mostrar que 3
(14)
1 (mod 14). Bom,
precisamos conhecer (14). Para isso, listemos os n umeros positivos menores que 14 e primos com 14:
1 3 5 9 11 13
Vamos agora multiplicar todos estes n umeros por a = 3 e calcular cada um deles modulo 14:
1 3 3 (mod 14)
3 3 9 (mod 14)
5 3 1 (mod 14)
9 3 13 (mod 14)
11 3 5 (mod 14)
13 3 11 (mod 14)
36
Vejam! Todos os n umeros positivos menores que 14 e primos com 14 apareceram novamente, so que em
outra ordem! Multiplicando todas estas (14) congruencias, temos
(1 3 5 9 11 13) 3
(14)
(3 9 1 13 15 11) (mod 14)
Como 1 3 5 9 11 13 e primo com 14 (estamos multiplicando n umeros sem fatores em comum com 14,
logo o produto continua sem fatores em comum com 14), este produto e inversvel modulo 14 e portanto
cancelando obtemos
3
(14)
1 (mod 14)
Lembra algo? Se voce disse Euler-Fermat, acertou!
3.2.3 No caso geral. . .
O que zemos? Primeiro, listamos os n umeros positivos a
1
, a
2
, . . . , a
(m)
menores que m e primos com
m. A multiplicamos cada um deles por a e, calculando tudo modulo m, obtivemos os mesmo n umeros
a
1
, a
2
, . . . , a
(m)
em outra ordem. Depois era so multiplicar tudo.
Ser a que este fenomeno e uma mera coincidencia? Obra do acaso? Ou seria consequencia do
Teorema 75 (Gira-Gira). Seja a primo com m. Multiplica c ao por a permuta os elementos de (Z/mZ)

e de Z/mZ. Explicitamente: se
T = a
1
, a
2
, . . . , a
(m)

e o conjunto dos n umeros positivos menores ou iguais a m e primos com m (de modo que T mod m =
(Z/mZ)

), ent ao
T = a a
1
mod m, a a
2
mod m, . . . , a a
(m)
mod m
Se
S = b
1
, b
2
, . . . , b
m

e um sistema completo de resduos m odulo m (de modo que S mod m = Z/mZ) ent ao
a b
1
, a b
2
, . . . , a b
m

tambem e um sistema completo de resduos m odulo m.


Demonstra c ao. Vamos demonstrar que multiplica c ao por a permuta os elementos de (Z/mZ)

e deixamos
o outro caso para o leitor.
Sejam
a
1
, a
2
, . . . , a
(m)
os (m) elementos de (Z/mZ)

. Temos que
a a
1
, a a
2
, . . . , a a
(m)
tambem sao elementos de (Z/mZ)

pois a (Z/mZ)

por hipotese e (Z/mZ)

e fechado por multiplica c ao.


Para mostrar que esta lista contem todos os elementos de (Z/mZ)

(em alguma ordem), basta mostrar que


eles sao todos distintos, ja que h a (m) deles.
Mas isto e facil: se houvesse uma repeti c ao a a
i
= a a
j
para dois elementos distintos a
i
e a
j
, cancelando
a obteramos a
i
= a
j
, um absurdo.
Vamos agora concluir a demonstra c ao do teorema de Euler-Fermat. Com a nota c ao acima, pelo gira-
gira, temos que
a a
1
a

1
(mod m)
a a
2
a

2
(mod m)
.
.
.
a a
(m)
a

(m)
(mod m)
37
onde os a

i
sao uma permuta c ao dos a
i
. Assim, multiplicando estas equa c oes, obtemos
a
(m)

1i(m)
a
i

1i(m)
a
i
(mod m)
Como cada a
i
e inversvel modulo m, o produto

1i(m)
a
i
tambem e inversvel modulo m. Pelo cancela-
mento, temos que a
(m)
1 (mod m), como desejado.
3.3 Utilizando o teorema de Euler-Fermat
Mostraremos aqui algumas aplica c oes do teorema de Euler-Fermat. A primeira e facilitar nossas contas.
Exemplo 76. Calcule o resto da divis ao de 2
2001
por 101.
Soluc ao. Temos que 2 e 101 sao primos entre si. Utilizando o teorema de Euler-Fermat, temos (101) = 100
e portanto, como 2001 = 100 20 + 1,
2
2001
=
_
2
100
_
20
2 1
20
2 (mod 101) = 2
2001
2 (mod 101)
Observe que podemos pensar isto da seguinte forma: como 2001 1 (mod (101)), temos 2
2001
2
1
(mod 101). Ou seja, as potencias de 2 modulo 101 sao peri odicas modulo (101).
Eis uma aplica c ao n ao exatamente do teorema de Euler-Fermat mas sim do gira-gira:
Exemplo 77. Mostre que
S = 1
6
+ 2
6
+ 3
6
+ 4
6
+ + 100
6
e divisvel por 101.
Soluc ao. Multiplicando S por 2
6
temos
2
6
S = (2 1)
6
+ (2 2)
6
+ + (2 100)
6
Pelo gira-gira, temos que 2 1, 2 2, . . . , 2 100 = 1, 2, . . . , 100, assim
2
6
S 1
6
+ 2
6
+ 3
6
+ 4
6
+ + 100
6
(mod 101) 2
6
S S (mod 101)
(2
6
1)S 0 (mod 101)
S 0 (mod 101)
ja que 2
6
1 = 63 e primo com 101.
Outra aplica c ao usa simplesmente a existencia do expoente magico (m).
Exemplo 78. Mostre que existe uma potencia de 3 cuja representa c ao decimal contem pelo menos 1000
zeros sucessivos.
Demonstra c ao. Como (3, 10
1001
) = 1, podemos escrever
3
(10
1001
)
1 (mod 10
1001
)
Assim, os ultimos 1001 dgitos de 3
(10
1001
)
sao 0000 . . . 1 (1000 zeros), o que termina o problema.
Exemplo 79. Mostre que existem innitos n umeros da forma 200 . . . 01 que s ao m ultiplos de 2001.
38
Soluc ao. Seja n o n umero de zeros em 200 . . . 01. Temos 200 . . . 01 = 2 100 . . . 00+1 = 2 10
n+1
+1. Assim,
basta encontrarmos innitos valores de n > 2 para os quais 2 10
n+1
+ 1 0 (mod 2001). Mas
2 10
n+1
+ 1 0 (mod 2001) 2 10
n+1
+ 1 2001 (mod 2001)
2 10
n+1
2000 (mod 2001)
10
n+1
10
3
(mod 2001)
Como n + 1 > 3, temos ent ao que encontrar innitos ns tais que
10
n2
1 (mod 2001) ()
Observe que a expressao acima e parecida com a do teorema de Euler-Fermat. De fato, como (10, 2001) =
1, temos, pelo teorema de Euler-Fermat, que existe um expoente (2001) tal que
10
(2001)
1 (mod 2001)
Mas a partir de um expoente, e facil encontrar innitos: elevando a congruencia anterior a um n umero
positivo k, temos
10
(2001)k
1 (mod 2001) ()
Assim, tomando n 2 = (2001)k n = (2001)k + 2, temos, a partir de (), o que queremos
em (). Assim, existem innitos n umeros da forma 200 . . . 01 (aqueles com n = (2001)k + 2 zeros) que sao
m ultiplos de 2001.
O proximo problema mostra como o teorema de Euler-Fermat e util para reconhecer quando n ao h a
solu c oes para certas equa c oes modulo m.
Exemplo 80. Mostre que se m e n s ao inteiros tais que 103 divide m
2
+ n
2
ent ao 103 divide m e n.
Soluc ao. Suponha por absurdo que n ,= 0. Ent ao m ,= 0 tambem: caso contrario, como m
2
+ n
2
0
(mod 103), teramos que m 0 (mod 103) = n
2
0 (mod 103) 103 [ n
2
103 [ n n 0
(mod 103) onde a equivalencia 103 [ n
2
103 [ n e valida pois 103 e primo.
Como n ,= 0 e 103 e primo, temos que n e inversvel em Z/103Z. Multiplicando a hipotese inicial
m
2
+ n
2
= 0 por (n
1
)
2
obtemos
m
2
+ n
2
= 0 = (m n
1
)
2
+ 1 = 0 (m n
1
)
2
= 1,
isto e, 1 e um quadrado perfeito modulo 103. Mas isto e impossvel: elevando ambos os lados a (103
1)/2 = 51, obtemos
(m n
1
)
2
= 1 = (m n
1
)
102
= 1
o que contradiz o teorema de Euler-Fermat (m n
1
)
102
= 1 (como m ,= 0, temos m n
1
,= 0, logo podemos
aplicar o teorema; por outro lado 1 , 1 (mod 103)).
O proximo exemplo ilustra a utiliza c ao do binomio de Newton juntamente com o teorema de Euler-
Fermat.
Exemplo 81. Dizemos que um n umero natural n tem a propriedade P se, e somente se,
n [ a
n
1 = n
2
[ a
n
1
para a Z.
(a) Mostre que todo n umero primo possui a propriedade P.
(b) Mostre que existem innitos n umeros compostos que possuem a propriedade P.
39
Soluc ao. Podemos escrever a propriedade P como
a
n
1 (mod n) = a
n
1 (mod n
2
)
(a) Suponha que n seja primo. Temos a
n
a (mod n) pelo pequeno teorema de Fermat, logo
a
n
1 (mod n) =a 1 (mod n) a = kn + 1 para algum k inteiro
Devemos mostrar que (kn + 1)
n
1 (mod n
2
) para n primo. Pela formula do binomio de Newton,
(kn + 1)
n
=
n

i=0
_
n
i
_
(kn)
i
1
ni
=
_
n
n
_
(kn)
n
+ +
_
n
2
_
(kn)
2
+
_
n
1
_
kn + 1
Tomando (kn +1)
n
mod n
2
, temos que todos as parcelas da forma
_
n
i
_
(kn)
i
para i 2 sao divisveis por
n
2
. Logo
(kn + 1)
n

_
n
1
_
(kn) + 1 = n kn + 1 1 (mod n
2
),
que e o que queramos demonstrar.
(b) Vamos encontrar innitos n umeros compostos que possuem a propriedade P. Os n umeros primos parecem
ser otimos exemplos, ent ao podemos tentar n = 2p onde p e primo mpar.
Novamente pelo pequeno teorema de Fermat a
p
a (mod p) e desta forma a
2p
a
2
(mod p). Portanto
a
n
1 (mod n) a
2p
1 (mod 2p) a
2
1 (mod p)
a
2
= kp + 1 para algum k inteiro
Alem disso, como n e par, de a
n
1 (mod n) temos tambem que a deve ser mpar.
Vamos mostrar agora que n = 2p tem a propriedade P. Devemos mostrar que a
2p
= (kp + 1)
p
1
(mod 4p
2
) para p > 2 primo. Isso e o mesmo que mostrar que: (i) (kp+1)
p
1 (mod 4); e (ii) (kp+1)
p
1
(mod p
2
).
Ja demonstramos (ii) no item (a). Mostremos (i). Lembrando que (kp+1)
p
= a
2p
= (a
p
)
2
e que a e mpar
(e consequentemente a
p
tambem), temos a
p
= 2 + 1 para algum inteiro e assim a
2p
= 4
2
+ 4 + 1 1
(mod 4), o que completa nossa demonstra c ao.
3.4 Exerccios
3.4.1 Basicos
36. Mostre que 2222
5555
+ 5555
2222
e divisvel por 7.
37. Calcule os dois ultimos dgitos de 7
2001
.
38. Mostre que
S = 1
48
+ 2
48
+ 3
48
+ + 96
48
e divisvel por 97.
39. Mostre que existem innitos n umeros da forma 19999 . . . 9991 que s ao m ultiplos de 1991.
40. Mostre que se m e n s ao inteiros tais que 1999 divide m
2
+ n
2
ent ao 1999 divide m e n.
40
3.4.2 Licao de casa
41. Mostre que
(a) n ao existe x inteiro tal que 43 [ x
2
+ 1
(b) n ao existe x inteiro tal que 103 [ x
3
2
(c) se 43 [ x
2
+ y
2
ent ao 43 [ x e 43 [ y
(d) se 103 [ x
3
2y
3
ent ao 103 [ x e 103 [ y
42. (a) Mostre que se p = 4k +3 (k Z) e primo e m e n s ao inteiros tais que p divide m
2
+n
2
, ent ao p
divide m e n.
(b) Mostre que se t = m
2
+ n
2
pode ser escrito como soma de dois quadrados perfeitos, ent ao todos os
fatores primos da forma 4k + 3, k inteiro, devem aparecer na fatora c ao de t com expoente par.
43. Mostre que para todo inteiro positivo n existe
(a) uma potencia de 7 cuja representa c ao decimal contem pelo menos n zeros sucessivos.
(b) uma potencia de 2 cuja representa c ao decimal contem pelo menos n zeros sucessivos.
Dica: no item (b), aplique o teorema de Euler-Fermat para a = 2 m odulo 5
k
.
44. Seja p um primo.
(a) Mostre que para 0 < k < p o coeciente binomial
_
p
k
_
e divisvel por p.
(b) Utilize o item anterior e o bin omio de Newton para provar, por induc ao em a, que a
p
a (mod p)
(pequeno Teorema de Fermat) para todo natural a.
45. Mostre que existem innitos n umeros da forma 100000 . . . 0001 que s ao divisveisl por 49.
3.4.3 Questoes de Provas
46. (IMO) Sejam a e b inteiros positivos tais que 15a+16b e 16a15b sejam quadrados perfeitos. Encontrar
o menor valor que pode assumir o menor destes quadrados.
Dica: considere (15a + 16b)
2
+ (16a 15b)
2
e analise m odulo um n umero conveniente.
47. Um n umero inteiro positivo e denominado n umero duplo se sua representa c ao decimal consiste de um
bloco de dgitos n ao iniciado por zero seguido imediatamente de um bloco identico. Por exemplo, 360360 e um
n umero duplo, mas 36036 n ao e. Mostre que existem innitos n umeros duplos que s ao quadrados perfeitos.
Dica: tente encontrar n umeros duplos com 6, 8 ou 10 dgitos!
48. Prove que o conjunto 2
k
3 [ k N contem um subconjunto innito cujos membros s ao primos dois
a dois.
41
Captulo 4
Ordem e Razes Primitivas
Neste captulo, estudaremos propriedades relacionadas ao grupo (Z/mZ)

. O conceito fundamental e o de
ordem de um inteiro a modulo m, que e uma especie de perodo para as potencias de a modulo m. Em
seguida, estudaremos os inteiros a que possuem perodo maximo (m), as chamadas razes primitivas.
4.1 Ordem e menor divide
Deni cao 82. Sejam a e m inteiros primos entre si. A ordem de a m odulo m (ou de a (Z/mZ)

),
denotada por ord
m
a, e o menor inteiro d > 0 tal que
a
d
1 (mod m) a
d
= 1
A ordem d = ord
m
a e uma especie de perodo para as potencias de a modulo m, pois como a
d
= 1,
temos que a
i+d
= a
i
e assim as potencias a
i
, i = 0, 1, 2, . . . se repetem de d em d. Este perodo e o perodo
mnimo pois tomamos d o menor possvel.
Exemplo 83. Para 2 Z/31Z, temos
2
0
= 1
2
1
= 2
2
2
= 4
2
3
= 8
2
4
= 16
2
5
= 1
2
6
= 2
2
7
= 4
2
8
= 8
2
9
= 16
. . .
de modo que ord
31
2 = 5.
Assim como o perodo mnimo de uma fun c ao trigonometrica divide qualquer outro perodo desta fun c ao,
temos aqui tambem o
Teorema 84 (Menor divide). Se a
t
1 (mod m) ent ao ord
m
a [ t. Em particular, ord
m
a divide (m).
Demonstra c ao. Seja d = ord
m
a e t = d q + r a divisao euclidiana de t por d, onde 0 r < d. Assim, como
a
d
1 (mod m), temos
a
t
1 (mod m)
_
a
d
_
q
a
r
1 (mod m) a
r
1 (mod m)
Se r > 0, temos um expoente menor que d tal que a
r
1 (mod m), absurdo. Logo r = 0 e portanto d divide
t. Por outro lado, como a
(m)
1 (mod m) por Euler-Fermat, devemos ter d [ (m) pelo que acabamos de
demonstrar.
42
Podemos utilizar o menor divide para facilitar o c alculo da ordem, evitando ter que listar todas as
potencias.
Exemplo 85. Determine ord
100
3.
Soluc ao. Para otimizar as contas, vamos quebrar o modulo 100 em potencias de primos. Note que
3
(25)
1 (mod 25) e 3
(4)
1 (mod 4). Temos (25) = 20 e (4) = 2, portanto temos que 3
20
1
(mod 25) e 3
20
1 (mod 4). Como 4 e 25 sao primos entre si e 3
20
1 e divisvel tanto por 4 como por 25,
temos que 3
20
1 (mod 100), logo ord
100
3 [ 20 pelo menor divide.
Para mostrar que ord
100
3 = 20, basta testar os divisores maximais de 20, que sao 20/2 = 10 e 20/5 = 4
(qualquer divisor proprio de 20 divide um dos dois n umeros). Fazendo as contas, temos que 3
10
49
(mod 100) e 3
4
81 (mod 100). Se ord
100
3 fosse algum divisor proprio de 20, uma destas congruencias
deveria ser igual a 1 modulo 100, o que n ao ocorre. Logo ord
100
3 = 20.
O conceito de ordem aparece em diversos problemas em Teoria dos N umeros. Vejamos dois exemplos
tpicos.
Exemplo 86. Sejam a e n dois inteiros positivos primos entre si. Mostre que n [ (a
n
1).
Soluc ao. Note que n e o menor inteiro positivo tal que a
n
1 (mod a
n
1). Assim, n e a ordem de a
modulo a
n
1 e o resultado segue diretamente do menor divide.
Exemplo 87. Mostre que se n e um inteiro maior que 1, ent ao n n ao divide 2
n
1.
Demonstra c ao. Suponhamos por absurdo que exista um valor de n > 1 tal que n divide 2
n
1. Seja p o
menor primo que divide n. Temos que, como n divide 2
n
1, ent ao p divide 2
n
1, isto e, 2
n
1 (mod p).
Seja d = ord
p
2.
Como (p) = p 1, temos pelo menor divide que
_

_
2
d
1 (mod p)
2
p1
1 (mod p)
2
n
1 (mod p)
=
_
d [ p 1
d [ n
Logo d e divisor comum de p 1 e n e portanto divide (p 1, n). Mas todos os divisores primos de p 1 sao
menores que p e portanto n ao aparecem na fatora c ao de n pela minimalidade de p. Logo (p 1, n) = 1 e o
unico valor possvel para d e 1.
Logo 2
1
1 (mod p) p [ 1, absurdo. Assim, n ao existe n > 1 tal que n divide 2
n
1.
4.2 Lema de Hensel
O lema de Hensel e um resultado que nos permite calcular a potencia exata de um primo que divide um certo
n umero. O lema em si e apenas uma simples aplica c ao do binomio de Newton, mas ele e uma ferramenta
muito util na resolu c ao de equa c oes diofantinas, especialmente as que envolvem variaveis no expoente.
Relembrando: para um primo p, escrevemos p

| m se p

e a maior potencia de p que divide m (ver


deni c ao 60).
Teorema 88 (Lema de Hensel). Seja p um primo mpar, a um inteiro e n um inteiro positivo. Sejam e
inteiros n ao negativos, com > 0.
(i) Se p

| n e p

| a 1 ent ao p
+
| a
n
1 (atenc ao, p deve dividir a 1 pois > 0! Mas note que p
n ao precisa dividir n)
(ii) Se n e mpar, p

| n e p

| a + 1 ent ao p
+
| a
n
+ 1 (mesma resalva do item (i)).
43
Demonstra c ao. Vamos demonstrar o item (i) e deixar o item (ii) como exerccio. Observe que n = p

k,
sendo que k n ao e m ultiplo de p. Primeiro provaremos o resultado para k = 1, ou seja, n = p

.
A demonstra c ao e por indu c ao sobre . Para = 0 o resultado e obvio. Suponha que o resultado e
valido para = t, ou seja, que a
p
t
1 = p
+t
m, com m n ao divisvel por p. Assim, a
p
t
= p
+t
m + 1.
Elevando os dois lados a p e utilizando binomio de Newton, obtemos
a
p
t+1
= (p
+t
m + 1)
p
=
_
p
0
_
1
p
+
_
p
1
_
1
p1
p
+t
m +
_
p
2
_
1
p2
(p
+t
m)
2
+ +
_
p
p
_
(p
+t
m)
p
a
p
t+1
1 = p
+t+1
(m + p v),
em que v e um inteiro. Logo, como m n ao e divisvel por p, temos que m + p v tambem n ao e m ultiplo de
p, o que conclui a demonstra c ao para o caso especial k = 1.
Se k > 1, basta observar que
a
p

k
1 =
_
a
p
_
k
1 = (a
p

1) (a
p

(k1)
+ a
p

(k2)
+ + a
p

+ 1) ()
Como a
p

1 (mod p) (pois p [ a 1 por hipotese) temos que


a
p

(k1)
+ a
p

(k2)
+ + a
p

+ 1 1 + 1 + + 1
. .
k uns
k (mod p)
e como k , 0 (mod p), temos que o segundo fator em () n ao e divisvel por p. Logo apenas o primeiro fator
em () contribui com fatores p e pelo caso especial ja provado temos que p
+
| a
n
1.
Exemplo 89. Encontre todos os inteiros n ao negativos x e y tais que
7
y
2 3
x
= 1
Demonstra c ao. Temos 2 3
x
= 7
y
1. Note que a maior potencia de 3 que divide 7 1 e 3. Seja 3
m
a maior
potencia de 3 que divide y. Ent ao, pelo lema de Hensel, a maior potencia de 3 que divide 7
y
1 e 3
m+1
.
Logo x = m + 1.
Agora vamos utilizar desigualdades. Como 3
m
divide y, 3
m
y, e assim
2 3
m+1
= 2 3
x
= 7
y
1 7
3
m
1.
o que impoe uma forte restri c ao sobre os possveis valores de m, pois para m sucientemente grande o lado
da direita e muito maior que o da esquerda, mas a desigualdade est a no sentido oposto!
Sendo t = 3
m
, temos 6t 7
t
1 7
t
1 + 6t, que e verdadeiro para t = 0 e t = 1 mas falso
para t > 1, pois nesse caso 7
t
= (6 + 1)
t
>
_
t
0
_
1
t
+
_
t
1
_
1
t1
6 = 1 + 6t. Assim, a unica possibilidade e
t = 1 m = 0, logo x = m + 1 = 1 e assim y = 1.
4.3 Razes Primitivas
Ja sabemos que ord
m
a [ (m).

E possvel que ord
m
a = (m)? E se isso acontecer, sera que isso e util? Este
fato e t ao util que n umeros com essa propriedade ganham um nome especial:
Deni cao 90. Dizemos que g e raiz primitiva de m quando ord
m
g = (m).
Um fato simples mas importante e que se g e uma raiz primitiva modulo m ent ao as potencias de g
geram todo o grupo (Z/mZ)

(da a nossa escolha da letra g). Precisamente, temos


44
Lema 91. Se g e raiz primitiva de m ent ao
(Z/mZ)

= 1, g, g
2
, . . . , g
(m)1

Ou seja, se (a, m) = 1 ent ao existe i tal que a g


i
(mod m).
Demonstra c ao. Como (Z/mZ)

tem (m) elementos, basta mostrar que dentre as (m) potencias 1, g, g


2
, . . . , g
(m)1
de g n ao h a duas repetidas modulo m. Mas se g
i
= g
j
para 0 i < j < (m) ent ao g
ji
= 1 com
0 < j i < (m), contradizendo o fato de que g tem ordem (m).
Parece promissor, n ao? Pena que nem todos os n umeros admitem razes primitivas. O seguinte teorema
e o principal resultado deste captulo:
Teorema 92 (Razes Primitivas). Os n umeros que admitem razes primitivas s ao 2, 4, p
n
e 2p
n
, sendo p
primo mpar.
A demonstra c ao deste teorema e longa e sera dada nas proximas se c oes. Vejamos antes alguns exemplos.
Exemplo 93. Seja p um primo. Mostre que
1
k
+ 2
k
+ + (p 1)
k
mod p =
_
0 se (p 1) k
p 1 se (p 1) [ k
Soluc ao. Se (p 1) [ k, temos que cada termo da soma acima e congruente a 1 modulo p e o resultado
segue. Suponha agora que (p 1) k e seja g uma raiz primitiva modulo p. Temos portanto
1
k
+ 2
k
+ + (p 1)
k
1 + g
k
+ g
2k
+ + g
(p2)k
(mod p)
Gostaramos de somar a PG modulo p acima. Podemos utilizar o mesmo truque da soma da PG usual:
sendo S = 1 +g
k
+g
2k
+ +g
(p2)k
, multiplicando pela razao g
k
e observando que g
(p1)k
1 (mod p)
temos
g
k
S g
k
+ g
2k
+ + g
(p1)k
(mod p)
g
k
S 1 + g
k
+ g
2k
+ + g
(p2)k
(mod p)
g
k
S S (mod p) (g
k
1)S 0 (mod p)
Como g e uma raiz primitiva e (p 1) k temos que g
k
1 , 0 (mod p), assim pelo cancelamento obtemos
S 0 (mod p), o que encerra a prova.
Exemplo 94. Mostre que 2 e uma raiz primitiva m odulo 3
k
para todo k 1.
Soluc ao. Temos que mostrar que ord
3
k 2 = (3
k
) = 2 3
k1
. A prova e por indu c ao em k. O resultado
e verdadeiro para a base k = 1. Suponha agora que o resultado valha para k e seja d = ord
3
k+1 2. Pelo
menor divide temos que d [ (3
k+1
) d [ 2 3
k
. Por outro lado, como 2
d
1 (mod 3
k+1
) = 2
d
1
(mod 3
k
) e ord
3
k 2 = 2 3
k1
por hipotese de indu c ao, temos tambem que 2 3
k1
[ d. Assim, so h a duas
possibilidades para d: ou d = 2 3
k1
ou d = 2 3
k
. Devemos mostrar que a primeira n ao ocorre, ou seja,
que 2
23
k1
, 1 (mod 3
k+1
).
Mas isto e um trabalho para o lema de Hensel! Pelo lema, temos que 3
k
| 2
23
k1
1, assim 2
23
k1
, 1
(mod 3
k+1
), o que encerra a prova.
45
O lema a seguir mostra como, a partir de uma raiz primitiva, encontrar todas as demais.
Lema 95. Seja g uma raiz primitiva m odulo m. Mostre que as razes primitivas de m s ao exatamente as
potencias g
i
de g com (i, (m)) = 1. Em particular, m possui ((m)) razes primitivas (se possuir alguma).
Soluc ao. Como qualquer elemento de (Z/mZ)

e uma potencia de g, temos que as razes primitivas de


m tambem se escrevem como g
i
para algum i e a quest ao se resume a provar que ord
m
g
i
= (m)
(i, (m)) = 1.
Seja d = (i, (m)) e t = ord
m
g
i
. Vamos provar que t = (m)/d, o que implica o lema. Primeiramente,
observe que como i/d Z temos
_
g
i
_
(m)/d
=
_
g
(m)
_
i/d
= 1
Assim, pelo menor divide temos que t divide (m)/d. Para mostrar que, reciprocamente, (m)/d divide
t, observe que como
_
g
i
_
t
= 1 g
it
= 1
e g e uma raiz primitiva, novamente pelo menor divide temos que (m) [ it
(m)
d
[
i
d
t. Como
(
(m)
d
,
i
d
) = 1, pela alternativa de primo temos que
(m)
d
[ t, completando a demonstra c ao.
4.4 Demonstracao do Teorema Principal
Vamos agora demonstrar o teorema acima. Come camos com uma pequena digress ao sobre polinomios sobre
corpos nitos.
4.4.1 Polinomios com coecientes em Z/pZ
Um corpo e um anel em que a multiplica c ao e comutativa e em que todo elemento n ao nulo possui inverso
multiplicativo. Por exemplo, Q, R e C sao corpos, mas Z n ao. Uma quest ao natural e: quando Z/mZ e um
corpo?
Lema 96 (Corpo Finito). Z/pZ e um corpo se, e somente se, p e primo.
Demonstra c ao. Se p e primo, ent ao todo elemento com exce c ao de 0 e inversvel, logo Z/pZ e um corpo.
Reciprocamente, se Z/pZ e um corpo, os n umeros 1, 2, . . . , p 1 devem ser todos inversveis modulo p, isto
e, primos com p. Isto implica que p e primo (c.f. demonstra c ao do teorema de Wilson).
Quando trabalhamos em um corpo K podemos fazer a divisao euclidiana de polinomios com coecientes
em K utilizando o algoritmo da chave e o fato de que qualquer coeciente n ao nulo ser inversvel. Por exemplo,
dividindo o polinomio x
3
+ 3x + 2 por 2x + 1 com coecientes em Z/5Z obtemos quociente 3x
2
+ x + 1 e
resto 1:
x
3
+0x
2
+3x+2 2x + 1
x
3
3x
2
3x
2
+ x + 1
2x
2
+3x+2
2x
2
x
2x+2
2x1
1
Teorema 97. Seja f(x) um polin omio com coecientes inteiros e de grau d e p um primo. Ent ao a con-
gruencia f(x) 0 (mod p) tem no m aximo d razes m odulo p, contando multiplicidades.
46
Demonstra c ao. Indu c ao em d. Para d = 0, n ao h a razes e para d = 1, o polinomio e da forma ax +b, a , 0
(mod p), cuja raiz e x = b a
1
mod p.
Seja f(x) de grau d e r uma raiz de f (caso f n ao tenha razes, o teorema est a demonstrado, pois d 0).
Ent ao, pelo teorema do resto, f(x) (xr)g(x) (mod p), sendo g de grau d1. Pela alternativa de primo,
temos que (x r)g(x) 0 (mod p) x r (mod p) ou g(x) 0 (mod p). Pela hipotese de indu c ao, g
tem no maximo d 1 razes, e o resultado segue.
Note que esse resultado n ao e valido para modulos compostos. Por exemplo, x
2
1 0 (mod 8)
tem 4 solu c oes, a saber, todos os mpares modulo 8.
4.4.2 Enm, a demonstracao!
Vamos provar o teorema 92 em varias partes.
Lema 98. Se m tem dois fatores primos mpares distintos p e q ent ao m n ao admite raiz primitiva.
Demonstra c ao. Se m admite dois fatores primos mpares distintos p e q ent ao podemos escrever m = a b,
com (a, b) = 1, a, b > 1, p [ a e q [ b. Note que p 1 [ (a) e q 1 [ (b). Em particular, (a) e (b)
sao ambos pares, ou seja, (a)/2 e (b)/2 sao ambos inteiros. Assim, sendo x primo com m, x
(a)
1
(mod a) = x
(a)(b)/2
1 (mod a). Analogamente, x
(a)(b)/2
1 (mod b). Como a e b sao primos entre
si, conclumos que x
(a)(b)/2
1 (mod ab), de modo que ord
m
x (a)(b)/2 = (ab)/2 = (m)/2 < (m);
ou seja, m n ao admite raiz primitiva.
Lema 99. 2 e 4 admitem raiz primitiva, mas 2
n
, n 3, n ao.
Demonstra c ao. Primeiro, 1 e 3 sao razes primitivas de 2 e 4, respectivamente. Seja x mpar. Temos x
2
1
(mod 8) e, para n 3,
x
2
n2
1 = (x
2
1)(x
2
+ 1)(x
4
+ 1) . . . (x
2
n3
+ 1)
tem pelo menos n fatores 2, ja que 2
3
[ x
2
1 e 2 [ x
2
i
+1, i = 1, 2, . . . , n 3. Logo x
2
n2
1 (mod 2
n
), ou
seja, ord
2
n x 2
n2
< (2
n
) para todo x mpar. Assim, 2
n
n ao admite razes primitivas para n 3.
Lema 100. Se 4 [ m e m > 4 ent ao m n ao admite raiz primitiva.
Demonstra c ao. Seja m = 2
k
, mpar e k 2. O caso = 1 ja foi estudado no lema anterior. Nos demais
casos, basta repetir a demonstra c ao do primeiro lema com 4 no lugar de p e q sendo um divisor de .
Lema 101. Todo primo mpar p admite raiz primitiva.
Demonstra c ao. Essa e a parte mais difcil do teorema e e aqui que utilizaremos o teorema sobre po-
lin omios mod p. Considere o seguinte algoritmo para encontrar uma raiz primitiva de qualquer primo p:
Algoritmo
1. Tome a = 2.
2. Seja d
a
= ord
p
a. Se d
a
= p 1, a e raiz primitiva de p; caso contrario, tome o menor n umero b que
n ao e congruente a algum a
i
mod p.
3. Seja d
b
= ord
p
b. Se d
b
= p 1, b e raiz primitiva de p; se n ao, tome m e n tais que (m, n) = 1, m [ d
a
,
n [ d
b
e mn = mmc(d
a
, d
b
) (por que eles existem?).
4. Troque a por c = a
da/m
b
d
b
/n
e volte ao passo 2.
47
Vamos provar que esse algoritmo funciona e, o mais importante, termina. Supondo que termine, o
algoritmo funciona porque ele so para quando encontramos uma raiz primitiva.
Agora, provemos que o algoritmo termina, o que e mais interessante. Primeiro note que d
b
n ao divide
d
a
, pois se dividisse teramos b
da
1 (mod p), o que n ao pode ocorrer pois a equa c ao x
da
1 (mod p)
admite no maximo d
a
solu c oes, que sao 1, a, a
2
, . . . , a
da1
, e b n ao e congruente a nenhum a
i
. Isso implica
mmc(d
a
, d
b
) > d
a
. Alem disso, seja k = ord
p
c. Ent ao (a
da/m
b
d
b
/n
)
k
1 (mod p). Elevando ambos os
membros por m, obtemos a
dak
b
d
b
mk/n
1 (mod p) b
d
b
mk/n
1 (mod p). Lembrando que d
b
= ord
p
b,
pelo menor divide d
b
[ d
b
mk/n n [ mk. Sendo (m, n) = 1, temos n [ k. Analogamente, m [ k e,
portanto mn [ k. Observando ainda que c
mn
1 (mod p), temos ord
p
c = mn = mmc(d
a
, d
b
) > d
a
. Isto quer
dizer quem a cada itera c ao do algoritmo a ordem do proximo valor aumenta. Portanto, em algum momento
iguala o seu maximo, que e p 1 (note que a escolha de b no algoritmo depende de ord
p
a ,= p 1).
Lema 102. Se g e raiz primitiva de p mas n ao de p
2
, ent ao g + p e raiz primitiva de ambos.
Demonstra c ao. Seja d = ord
p
2 g. Ent ao
g
d
1 (mod p
2
) = g
d
1 (mod p) = p 1 [ d
Logo, como d ,= p(p 1) e d [ p(p 1), d = p 1, isto e, g
p1
1 (mod p
2
).
Basta agora demonstrarmos que (g + p)
p1
, 1 (mod p
2
). E esse e um trabalho para o binomio de
Newton! Temos (g + p)
p1
g
p1
+
_
p1
1
_
g
p2
p (mod p
2
) (voce consegue ver por que n ao precisamos
escrever os demais termos do binomio de Newton?). Substituindo g
p1
1 (mod p
2
) e desenvolvendo
obtemos (g + p)
p1
1 + (p 1)pg
p2
1 pg
p2
(mod p
2
), que n ao e 1, pois g
p2
, 0 (mod p). Logo
g + p e raiz primitiva de p
2
(e de p tambem!)
Lema 103. Se g e raiz primitiva de p e p
2
ent ao e raiz primitiva de p
n
, e portanto p
n
admite raiz primitiva.
Demonstra c ao. Indu c ao sobre n. A base de indu c ao (n = 1 e n = 2) est a na hipotese. Suponha que g seja
raiz primitiva de p
n1
. Seja d = ord
p
n g. Ent ao
g
d
1 (mod p
n
) = g
d
1 (mod p
n1
) = ord
p
n1 g [ d = p
n2
(p 1) [ d
Como d [ (p
n
) d [ p
n1
(p 1), temos d = p
n2
(p 1) ou d = p
n1
(p 1). Para eliminar o primeiro
caso, vamos usar o lema de Hensel.
Note que como p [ g
p1
1 e p
2
n ao divide g
p1
1 (caso contrario, g n ao seria raiz primitiva de p
2
),
p | g
p1
1. Assim, pelo lema de Hensel, a maior potencia de p que divide g
p
n2
(p1)
1 = (g
p1
)
p
n2
1
e p
1+n2
= p
n1
. Assim, p
n
n ao divide g
p
n2
(p1)
1, o que e equivalente a g
p
n2
(p1)
, 1 (mod p
n
). Logo
ord
p
n g = p
n1
(p 1) = (p
n
) e, portanto, g e raiz primitiva de p
n
.
Lema 104. 2p
n
admite raiz primitiva.
Demonstra c ao. Seja g uma raiz primitiva de p
n
. Note que (2p
n
) = (p
n
) (verique!) e considere g ou
g + p
n
, o que for mpar, e denote-o por h. Sendo d a ordem de h, temos h
d
1 (mod 2p
n
) = h
d
1
(mod p
n
) = (p
n
) [ d. Assim, d = (2p
n
).
Ufa! Com isto, o teorema est a provado!
4.5 Exerccios
4.5.1 Basicos
49. Encontre a ordem de
(a) 11 mod 190 (b) 7 mod 123
(c) 5 mod 1024 (d) 2 mod 343
48
50. Encontre todas as razes primitivas de
(a) 11 (b) 23
(c) 10 (d) 50
51. Mostre que
(a) 2 e uma raiz primitiva de 11;
(b) 7 e uma raiz primitiva de 239;
(c) 2 e uma raiz primitiva de 5
k
para todo k.
52. Sejam x e y inteiros positivos. Prove que existem inteiros positivos m e n tais que m [ x, n [ y,
(m, n) = 1 e mn = mmc(x, y).
53. Prove o item (ii) do lema de Hensel.
4.5.2 Licao de Casa
54. (IMO) Os tres ultimos dgitos de 1978
m
s ao iguais aos tres ultimos dgitos de 1978
n
(1 m < n,
m, n N). Determine m e n tais que m + n seja mnimo. (Cuidado! A resposta n ao e 1 + (1000)!!)
55. Mostre que se p e um primo tal que p [ 2
2
n
+1 ent ao p 1 (mod 2
n+1
). Utilize este fato para encontrar
um fator primo de 2
2
5
+ 1.
56. Sendo k 2 e n
1
, n
2
, . . . , n
k
1 n umeros naturais tais que n
2
[2
n1
1, n
3
[2
n2
1, . . . , n
k
[2
n
k1
1 e
n
1
[2
n
k
1, mostre que n
1
= n
2
= = n
k
= 1.
57. (OBM) Encontre todas as fun c oes f : Z
>0
R tais que, para todos x, y inteiros n ao negativos,
f(x)f(y) = f(xy) e f(x + 1019) = f(x)
58. Seja f(x) um polin omio de coecientes inteiros com grau d. Prove que a congruencia f(x) 0 (mod p),
p primo, tem d solu c oes n ao congruentes entre si mod p se, e somente se, na divis ao euclidiana de x
p
x
por f(x), x
p
x = f(x)q(x) + r(x), o resto r(x) tiver todos os seus coecientes m ultiplos de p.
59. Sendo k um inteiro positivo dado, encontre todos os inteiros positivos n tais que 7
n
+1 e m ultiplo de 5
k
.
60. H a outra maneira de provar que p admite raiz primitiva. Como? Siga os itens!
(a) Prove que

d|n
(d) = n. Dica: conte de duas maneiras a quantidade de pares (x, d) em que (x, n) = d.
(b) Prove que se d [ p 1 a congruencia x
d
1 (mod p) tem exatamente d solu c oes distintas mod p.
(c) Seja d um divisor de p 1 e r(d) a quantidade de n umeros mod p com ordem igual a d. Prove que
r(d) (d).
(d) Prove que, na verdade, r(d) = (d). Em particular, p admite (p 1) razes primitivas.
4.5.3 Questoes de Prova
61. Encontre todos os n umeros inteiros positivos n tais que x
25
x (mod n) para todo inteiro x.
62. (IMO) Determine se existe um inteiro n com 2000 divisores primos distintos tal que n divide 2
n
+ 1.
49
63. Encontre todos os inteiros m, n e p, onde p e um primo mpar tais que
p
m
n
p
= 1
Dica: tente encontrar uma fatora c ao e calcule o mdc dos dois fatores da fatora c ao. Depois use o binomio de
Newton para concluir o problema.
64. (China) Encontre todos os inteiros n ao negativos x, y, z e w tais que
2
x
3
y
5
z
7
w
= 1
65. (IMO) Determine todos os inteiros n 1 tais que (2
n
+ 1)/n
2
seja inteiro.
66. (IMO) Determine todos os pares (n, p) de inteiros estritamente positivos tais que p e primo, n 2p e
(p 1)
n
+ 1 e divisvel por n
p1
.
67. (Banco IMO) Encontre todas as ternas (a, m, n) de inteiros positivos tais que a
m
+ 1 divide (a + 1)
n
.
68. (Banco IMO) Seja a > 1 e m > n inteiros positivos. Prove que se a
m
1 e a
n
1 tem os mesmos
divisores primos ent ao a + 1 e uma potencia de 2.
69. (IMO Shortlist) Seja p > 3 um primo. Prove que existe a com 1 a < p 1 tal que a
p1
1 e
(a + 1)
p1
1 n ao s ao divisveis por p
2
.
50

Potrebbero piacerti anche